You are on page 1of 47

Page 1 of 47

PTS 2024 | B4 | L1 Test 1 - Solutions |

Q.1)
Ans) b
Exp) Option b is the correct answer
Statement 1 is incorrect: It is not the Charter Act of 1813 but the Charter Act of 1833 that ended the
activities of the East India Company as a commercial body and made East India Company a purely
administrative body. Important feature of the Charter Act of 1813 are as follows:
1) It abolished the trade monopoly of the company in India except in trade in tea and trade with China.
2) It asserted the sovereignty of the British Crown over the Company’s territories in India.
3) It allowed the Christian missionaries to come to India for the purpose of enlightening the people.
Statement 2 is correct: It is true that the Charter Act of 1833 deprived the Governor of Bombay and
Madras of their legislative powers. The Governor-General of India was given exclusive legislative powers
for the entire British India. Also the act made the Governor-General of Bengal as the Governor General of
India and vested in him all civil and military powers
Statement 3 is correct: It is true that the Charter Act of 1853 for the first time successfully introduced
an open competition system of selection and recruitment of civil servants. Previously the Charter Act of
1833 attempted to introduce a system of open competition for selection of civil servants but this
provision was negated after opposition from the Court of Directors.
Source: Laxmikanth - Chapter 1 (Historical Background)

Q.2)
Ans) a
Exp) Option a is the correct answer
A federal government is a system of dividing up power between a central National government and local
state governments such as State Governments and Local Self Governments.
Statement I and II are correct: The term ‘Federation’ has nowhere been used in the Constitution of
India. Article 1 describes India as a ‘Union of States’ which implies two things:
1) Indian Federation is not the result of an agreement by the states
2) No state has the right to secede from the federation (this explains why Article I defines India as a
Union of States). Hence Statement II is the correct explanation of Statement I.
Source: Laxmikanth - Chapter 3 (Salient Features of the Constitution)

Q.3)
Ans) c
Exp) Option c is the correct answer.
The Constitution of India deals with the citizenship from Articles 5 to 11 under Part II. The Constitution
empowers the Parliament to enact a law to provide for such matters and any other matter relating to
citizenship. Accordingly, the Parliament has enacted the Citizenship Act (1955), which has been amended
from time to time.
Option c is correct: Citizenship is a relation between state and an individual that entails specific legal
rights and duties that both individual and state owes towards each other. For example, it is the
responsibility of the State to look after the welfare of Indian citizens and individuals owe duties to states
such as to respect the national flag, national anthem etc.
A citizen is a participatory member of a political community. Citizenship is gained by meeting the legal
requirements of a national, state, or local government. A nation grants certain rights and privileges to its
citizens. In return, citizens are expected to obey their country's laws and defend it against its enemies.
Option d is incorrect: Living in a country does not mean that a person is necessarily a citizen of that
country. Citizens of one country who live in a foreign country are known as aliens for example USA

Forum Learning Centre: Delhi - 2nd Floor, IAPL House, 19 Pusa Road, Karol Bagh, New Delhi - 110005 | Patna - 2nd floor, AG Palace, E Boring Canal
Road, Patna, Bihar 800001 | Hyderabad - 1st & 2nd Floor, SM Plaza, RTC X Rd, Indira Park Road, Jawahar Nagar, Hyderabad, Telangana 500020
9311740400, 9311740900 | https://academy.forumias.com | admissions@forumias.academy | helpdesk@forumias.academy
Page 2 of 47

PTS 2024 | B4 | L1 Test 1 - Solutions |


citizens living in India are referred to as aliens of India. Hence this statement does not best reflect the
meaning of Citizenship.
Source: https://csc.wayne.edu/what-is-citizenship#:~:text=realm%20of%20citizenship.-
,Definition%20of%20citizenship,as%20a%20synonym%20for%20nationality.

Q.4)
Ans) b
Exp) Option b is the correct answer.
The Communal Award, also known as the MacDonald Award, was a scheme for representation of
minorities. It was announced in August 1932 by Ramsay MacDonald, the British Prime Minister.
Statement 1 is correct. The Communal Award was based on the findings of the Indian Franchise
Committee also called the Lothian Committee.
Simon Commission rejected the proposal of a separate electorate for the depressed classes; however, it
retained the concept of reserving seats.
Statement 2 is incorrect. The Indian Councils Act 1909 established a distinct electorate for Muslims,
which was later expanded to include Sikhs, Indian Christians, Anglo-Indians, and Europeans by the
Government of India Act 1919 (not by Communal Award). The Communal Award not only continued
separate electorates for the Muslims, Sikhs, Indian Christians, Anglo-Indians and Europeans but also
extended it to the depressed classes (Scheduled Castes).
Statement 3 is correct. Mahatma Gandhi was distressed over this extension of the principle of communal
representation to the depressed classes and undertook fast unto death in Yerawada Jail (Poona) to get the
award modified. This led to an agreement between the leaders of the Congress and the depressed classes
which is known as Poona Pact. The pact retained the Hindu joint electorate and gave reserved seats to
the depressed classes.
Source: Indian Polity, M. Laxmikanth, 6th Edition, Chapter-1

Q.5)
Ans) b
Exp) Option b is the correct answer.
Microsoft Research is helping preserve 'rare' Indian languages with its Project ELLORA in India.
Microsoft researchers under this project have been working toward creating digital ecosystems for
Indian languages that do not have enough presence online.
The main goal of the Project is to impact underserved communities through enabling language
technology by creating economic opportunities, building technological skills, enhancing education and
preserving local languages and cultures for future generations. Microsoft Research (MSR) has chosen to
focus on three languages namely; Gondi, Mundari and Idu Mishmi.
Source: https://indianexpress.com/article/technology/how-microsofts-project-ellora-is-helping-
small-languages-like-gondi-mundari-become-eloquent-for-the-digital-world-8413587/

Q.6)
Ans) d
Exp) Option d is the correct answer
Option a is correct: It is true that the rule of law ensures supremacy of law, Constitutionalism ensures
supremacy of the constitution. Rule of law means no one is above the law thus ensuring supremacy of
law (rather than individual) and Constitutionalism by requiring the government to follow basic
procedures ensures constitutional supremacy.

Forum Learning Centre: Delhi - 2nd Floor, IAPL House, 19 Pusa Road, Karol Bagh, New Delhi - 110005 | Patna - 2nd floor, AG Palace, E Boring Canal
Road, Patna, Bihar 800001 | Hyderabad - 1st & 2nd Floor, SM Plaza, RTC X Rd, Indira Park Road, Jawahar Nagar, Hyderabad, Telangana 500020
9311740400, 9311740900 | https://academy.forumias.com | admissions@forumias.academy | helpdesk@forumias.academy
Page 3 of 47

PTS 2024 | B4 | L1 Test 1 - Solutions |

Option b is correct: The basic principles of Constitutionalism such as the separation of powers, judicial
review, the prohibition of retroactive legislation ensures not only the implementation of rule of law in
letter but also in spirit and these principles enable law to be fair.
Option c is correct: Without rule of law, upholding the principles of Constitutionalism is not possible.
Only when the supremacy of the rule of law is established, can supremacy of the constitution
(constitutionalism) exist. Constitutionalism additionally requires effective laws and their enforcement to
provide structure to its framework.
Option d is incorrect: Both rule of law and constitutionalism are related ideas about how the powers of
government and of state officials are to be limited. Thus, the Rule of law does not limit the power of the
Government is not correct.
Source:
https://www.legalservicesindia.com/article/1699/Constitutionalism.html#:~:text=be%20no%20constit
utionalism.-,Constitutionalism,-And%20Rule%20of

Q.7)
Ans) a
Exp) Option a is the correct answer
Statement 1 is correct: It is true that the ninth schedule was added by the First Constitutional
Amendment Act, 1951. The first Amendment added Article 31A and 31B to the constitution which ensured
certain laws were valid even if it violated the fundamental rights of citizens.
Statement 2 is incorrect: The Ninth Schedule of the Constitution contains a list of central and state
laws. Currently, 284 such laws are shielded from judicial review. Hence the schedule includes only laws
made by the Parliament is not correct.
Statement 3 is incorrect: Originally, Article 31B saves the acts and regulations included in the Ninth
Schedule from being challenged and invalidated on the ground of contravention of any of the
fundamental rights. However, in I.R. Coelho case (2007), the Supreme Court ruled that the laws placed
under the Ninth Schedule after April 24, 1973, are open to challenge in court if they violate fundamentals
rights guaranteed under Articles 14, 15, 19 and 21 or the ‘basic structure’ of the constitution.
Source: Laxmikanth - Chapter 7 (Fundamental Rights)
https://indianexpress.com/article/explained/ninth-schedule-of-the-constitution-explained-6265890/

Q.8)
Ans) d
Exp) Option d is the correct answer.
Option a is incorrect: Article 14 provides for equality before Law and equal Protection of laws. It states
that the State shall not deny to any person equality before the law or the equal protection of the laws
within the territory of India. Article 14 does not guarantee the right to free legal aid to individuals.
Option b is incorrect: Article 19 guarantees to all citizens the six rights and some of them are (i) Right to
freedom of speech and expression. (ii) Right to assemble peaceably and without arms etc. Article 19 does
not guarantee the right to free legal aid to individuals.
Option c is incorrect: Article 20 grants protection against arbitrary and excessive punishment to an
accused person. For example, it states that No person shall be convicted of any offence except for
violation of a law in force at the time of the commission of the offence. Hence Article 20 does not
guarantee the right to free legal aid.
Option d is correct: Article 21 declares that no person shall be deprived of his life or personal liberty
except according to procedure established by law. The Supreme court held that right to free legal aid is a

Forum Learning Centre: Delhi - 2nd Floor, IAPL House, 19 Pusa Road, Karol Bagh, New Delhi - 110005 | Patna - 2nd floor, AG Palace, E Boring Canal
Road, Patna, Bihar 800001 | Hyderabad - 1st & 2nd Floor, SM Plaza, RTC X Rd, Indira Park Road, Jawahar Nagar, Hyderabad, Telangana 500020
9311740400, 9311740900 | https://academy.forumias.com | admissions@forumias.academy | helpdesk@forumias.academy
Page 4 of 47

PTS 2024 | B4 | L1 Test 1 - Solutions |

part of Article 21. The court also held that right against solitary confinement and right to speedy trial as
part of Article 21.
Source: Laxmikanth - Chapter 7 (Fundamental Rights)

Q.9)
Ans) d
Exp) Option d is the correct answer.
Originally, the Right to Property was one of the seven Fundamental Rights under Part III of the
Constitution. It was dealt by Article 19(1) (f) and Article 31. However, the Right to Property was deleted
from the list of Fundamental Rights by the 44th Amendment Act of 1978. Instead, it is now made a legal
right under Article 300-A in Part XII of the Constitution.
Statement I is incorrect: Article 226 gives High Courts the ability to issue instructions, orders, and writs
to any person or authority, including the government to enforce legal as well as fundamental rights. An
individual can approach the high court directly in case of violation of his/her right to property as right
to property is a legal right in India.
Statement II is correct: It is true that the 44th Constitutional Amendment Act of 1978 removed the right
to property from Part III of the constitution (Part III enumerates fundamental rights). A new provision,
Article 300-A, was added to the constitution, which provided that “no person shall be deprived of his
property save by authority of law”.
Source: Laxmikanth - Chapter 7 (Fundamental Rights)

Q.10)
Ans) c
Exp) Option c is the correct answer.
Recent studies came out with the fact that Immune imprinting in the bodies, might be making new
boosters’ dose of COVID-19 far less effective than expected.
Option c is correct. Immune imprinting is a tendency of the body to repeat its immune response based
on the first variant it encountered — through infection or vaccination — when it comes across a newer
or slightly different variant of the same pathogen.
Imprinting acts as a database for the immune system, helping it put up a better response to repeat
infections. After the body is exposed to a virus for the first time, it produces memory B cells that circulate
in the bloodstream and quickly produce antibodies whenever the same strain of the virus infects again.
The problem occurs when a similar, not identical, variant of the virus is encountered by the body. In such
cases, the immune system, rather than generating new B cells, activates memory B cells, which produce
cross-reactive antibodies that bind to features found in both the old and new strains. These cross-
reactive antibodies offer some protection but are not as effective as the ones produced by B cells when
the body first encountered the original virus.
Source: https://indianexpress.com/article/explained/explained-health/immune-imprinting-boosters-
effective-explained-8397721/

Q.11)
Ans) c
Exp) Option c is the correct answer.
The Fundamental Rights are guaranteed by the Constitution to all persons without any discrimination.
They uphold the equality of all individuals, the dignity of the individual, the larger public interest and
unity of the nation.

Forum Learning Centre: Delhi - 2nd Floor, IAPL House, 19 Pusa Road, Karol Bagh, New Delhi - 110005 | Patna - 2nd floor, AG Palace, E Boring Canal
Road, Patna, Bihar 800001 | Hyderabad - 1st & 2nd Floor, SM Plaza, RTC X Rd, Indira Park Road, Jawahar Nagar, Hyderabad, Telangana 500020
9311740400, 9311740900 | https://academy.forumias.com | admissions@forumias.academy | helpdesk@forumias.academy
Page 5 of 47

PTS 2024 | B4 | L1 Test 1 - Solutions |

Statement 1 is correct: Fundamental Rights are not absolute but qualified. The state can impose
reasonable restrictions on them. However, whether such restrictions are reasonable or not is to be
decided by the courts. Thus, they strike a balance between the rights of the individual and those of
society as a whole, between individual liberty and social control.
Statement 2 is incorrect: State cannot impose restrictions on all the fundamental rights on the
grounds of Security of State. The State can impose reasonable restrictions on the exercise of the
freedom of speech and expression on the grounds of security of the state.
Source: Laxmikanth chapter 8 - Fundamental Rights, page 239

Q.12)
Ans) a
Exp) Option a is the correct answer.
The Directive Principles of State Policy are enumerated in Part IV of the Constitution from Articles 36 to
51. The Directive Principles along with the Fundamental Rights contain the philosophy of the Constitution
and is the soul of the Constitution.
Statement 1 is incorrect: The Directive Principles resemble the ‘Instrument of Instructions’
enumerated in the Government of India Act of 1935. Instrument of instructions were issued to the
Governor-General and to the Governors of the colonies of India by the British Government under the
Government of India Act of 1935. Directive Principles is merely another name for the instrument of
instructions. The only difference is that they are instructions to the legislature and the executive.
Statement 2 is correct: The framers of the Constitution borrowed the idea of Directive Principles of
State Policy (DPSP) from the Irish Constitution of 1937, which had copied it from the Spanish
Constitution.
Statement 3 is incorrect: So far, DPSP has been amended four times. The 42nd Amendment Act of 1976
added four new Directive Principles to the original list. The 44th Amendment Act of 1978 added one more
Directive Principle. The 86th Amendment Act of 2002 changed the subject-matter of Article 45 and made
elementary education a fundamental right under Article 21 A. The 97th Amendment Act of 2011 added a
new Directive Principle relating to cooperative societies.
Source: Laxmikanth chapter 8 - Directive Principles of State Policy, page 183.

Q.13)
Ans) c
Exp) Option c is the correct answer.
The original constitution contained only the fundamental rights and not the fundamental duties. In 1976,
fundamental duties were added in the Constitution. The Fundamental Duties in the Indian Constitution
are inspired by the Constitution of erstwhile USSR.
Statement 1 is correct: In 1976, the Congress Party set up the Sardar Swaran Singh Committee to make
recommendations about fundamental duties, the need and necessity of which was felt during the
operation of the internal emergency (1975–1977). The committee recommended the inclusion of a separate
chapter on fundamental duties in the Constitution. The Congress Government at the Centre accepted
these recommendations.
Statement 2 is incorrect: The 42nd Constitutional Amendment Act in 1976 added a new part, namely,
Part IVA to the Constitution. This new part consists of only one Article, that is, Article 51A, which for the
first time specified a code of ten fundamental duties of the citizens.
Statement 3 is correct: Unlike some of the Fundamental Rights which extend to all persons whether
citizens or foreigners, the Fundamental Duties are confined to citizens only and do not extend to
foreigners.

Forum Learning Centre: Delhi - 2nd Floor, IAPL House, 19 Pusa Road, Karol Bagh, New Delhi - 110005 | Patna - 2nd floor, AG Palace, E Boring Canal
Road, Patna, Bihar 800001 | Hyderabad - 1st & 2nd Floor, SM Plaza, RTC X Rd, Indira Park Road, Jawahar Nagar, Hyderabad, Telangana 500020
9311740400, 9311740900 | https://academy.forumias.com | admissions@forumias.academy | helpdesk@forumias.academy
Page 6 of 47

PTS 2024 | B4 | L1 Test 1 - Solutions |

Statement 4 is correct: Fundamental Duties help the courts in examining and determining the
constitutional validity of a law. In 1992, the Supreme Court ruled that in determining the
constitutionality of any law, if a court finds that the law in question seeks to give effect to a fundamental
duty, it may consider such law to be 'reasonable’ in relation to Article 14 (equality before law) or Article 19
(six freedoms) and thus save such law from unconstitutionality.
Source: Laxmikanth chapter 9 - Fundamental Duties, page 264

Q.14)
Ans) d
Exp) Option d is the correct answer.
In the Kesavananda Bharti case, the Constitutional Bench of the Supreme Court ruled that Parliament
could amend any part of the Constitution so long as it did not alter or amend the basic structure or
essential features of the Constitution. From the various judgements, the following have emerged as the
elements of the basic features of the Constitution-
1) Supremacy of the Constitution
2) Sovereign, democratic and republican nature of the Indian polity
3) Secular character of the Constitution
4) Separation of powers between the legislature, the executive and the judiciary
5) Federal character of the Constitution
6) Unity and integrity of the nation
7) Welfare state (socio-economic justice)
8) Judicial review
9) Freedom and dignity of the individual
10) Parliamentary system
11) Rule of law
12) Harmony and balance between Fundamental Rights and Directive Principles
13) Principle of equality
14) Free and fair elections
15) Independence of Judiciary
16) Limited power of Parliament to amend the Constitution
17) Effective access to justice
18) Principles (or essence) underlying fundamental rights.
19) Powers of the Supreme Court under Articles 32, 136, 141 and 1426
20) Powers of the High Courts under Articles 226 and 2277
In the Kesavananda Bharati case (1973), the Supreme Court upheld the validity of the 24th Amendment Act
(1971) and stated that Parliament is empowered to abridge or take away any of the Fundamental Rights.
At the same time, it laid down a new doctrine of the ‘basic structure’ (or ‘basic features’) of the
Constitution. It ruled that the constituent power of Parliament under Article 368 does not enable it to
alter the ‘basic structure’ of the Constitution. This means that the Parliament cannot abridge or take
away a Fundamental Right that forms a part of the ‘basic structure’ of the Constitution.
Source: Polity by Laxmikanth 5th edition chapter-11 page-11.2, 11.3

Q.15)
Ans) b
Exp) Option b is the correct answer.

Forum Learning Centre: Delhi - 2nd Floor, IAPL House, 19 Pusa Road, Karol Bagh, New Delhi - 110005 | Patna - 2nd floor, AG Palace, E Boring Canal
Road, Patna, Bihar 800001 | Hyderabad - 1st & 2nd Floor, SM Plaza, RTC X Rd, Indira Park Road, Jawahar Nagar, Hyderabad, Telangana 500020
9311740400, 9311740900 | https://academy.forumias.com | admissions@forumias.academy | helpdesk@forumias.academy
Page 7 of 47

PTS 2024 | B4 | L1 Test 1 - Solutions |

The Financial Services Institutions Bureau (FSIB), the headhunter for directors of state-owned banks
and financial institutions, recently recommended names for the posts of managing directors of Bank of
Baroda and Bank of India.
Statement 1 is correct: Financial Services Institutions Bureau (FSIB) is a government body set up under
the Department of Financial Services. It replaced the Bank Board’s Bureau (BBB) in 2022.
Statement 2 is incorrect: FSIB would be headed by a chairman, a central government nominee.
The board would comprise the Secretaries of the Department of Financial Services, the chairman of
IRDAI, and a deputy governor of the RBI. Additionally, it will have three part-time members who are
experts in banking and three more from the insurance sector.
Statement 3 is incorrect: The chairperson of FSIB is not the member of Monetary Policy Committee.
MPC comprises six members - three officials of the Reserve Bank of India and three external members
nominated by the government of India. The role of FSIB is to recommend names for the posts of for
directors of state-owned banks and financial institutions.
Statement 4 is correct: The Primary role of FSIB:
1) To identify manpower capabilities and ensure proper selection of talent for senior positions at
financial institutions owned by the government.
2) It is entrusted with making recommendations for the appointment of full-time directors and non-
executive chairman of state-run financial services institutions.
3) The final decision on the FSIB recommendation would be taken by the Appointments Committee of
the Cabinet headed by the Prime Minister.
Source: https://www.thehindu.com/business/fsib-recommends-names-for-md-posts-of-bank-of-
baroda-bank-of-india/article66378959.ece

Q.16)
Ans) c
Exp) Option c is the correct answer.
The Constitution had a socialist content in the form of certain Directive Principles of State Policy even
before the term was added by the 42nd Amendment in 1976. In other words, what was hitherto implicit in
the Constitution has now been made explicit.
Statement 1 is correct: Indian brand of socialism is a ‘democratic socialism’ and not a ‘communistic
socialism’.
1) Communistic socialism’ (also known as ‘state socialism’) involves the nationalisation of all means of
production and distribution and the abolition of private property.
2) Democratic socialism, on the other hand, holds faith in a ‘mixed economy’ where both public and
private sectors co-exist side by side
Directive Principles of State Policy (DPSPs) of Indian Constitution reflect the socialistic features. For
example, right to adequate means of livelihood for all citizens, Promote equal justice and free legal aid to
the poor, etc.
Indian constitution does not promote core capitalistic values. For example, the Right to Property as
Fundamental Right was abrogated by the 44th Constitutional Amendment Act.
Statement 2 is incorrect: The new economic policy (1991) of liberalisation, privatisation and globalisation
has diluted the socialist credentials of the Indian State. Hence, statement 2 is incorrect.
Source: Ch-4, Preamble, Laxmikanth

Forum Learning Centre: Delhi - 2nd Floor, IAPL House, 19 Pusa Road, Karol Bagh, New Delhi - 110005 | Patna - 2nd floor, AG Palace, E Boring Canal
Road, Patna, Bihar 800001 | Hyderabad - 1st & 2nd Floor, SM Plaza, RTC X Rd, Indira Park Road, Jawahar Nagar, Hyderabad, Telangana 500020
9311740400, 9311740900 | https://academy.forumias.com | admissions@forumias.academy | helpdesk@forumias.academy
Page 8 of 47

PTS 2024 | B4 | L1 Test 1 - Solutions |

Q.17)
Ans) c
Exp) Option c is the correct answer.
Articles 214 to 231 in Part VI of the Constitution deal with the organisation, independence, jurisdiction,
powers, procedures and so on of the high courts.
Option a is correct: The salaries, allowances, privileges, leave and pension of the judges of a high court
are determined from time to time by the Parliament.
Option b is correct: The High Court Judges (Salaries and Conditions of Service) Act, 1954 regulates the
salaries and conditions of service of the judges of the High Courts. While The Supreme Court Judges
(Salaries and Conditions of Service) Act, 1958 regulate the salaries and conditions of service of the judges
of the Supreme Court.
Option c is incorrect: The salaries and allowances of the judges of a high court cannot be varied to their
disadvantage after their appointment except during a financial emergency. Hence, reduction in salaries
and allowances of the judges of a high court is possible during financial emergency.
Option d is correct: The salaries and allowances of the judges, the salaries, allowances and pensions of
the staff as well as the administrative expenses of a high court are charged on the consolidated fund of
the state. Thus, they are non-votable by the state legislature (though they can be discussed by it). It
should be noted here that the pension of a high court judge is charged on the Consolidated Fund of
India and not the state.
Source: Ch-34, High Court, Laxmikanth
https://prsindia.org/billtrack/the-high-court-and-the-supreme-court-judges-salaries-and-
conditions-of-service-amendment-bill-2017

Q.18)
Ans) a
Exp) Option a is the correct answer.
The Directive Principles of State Policy are enumerated in Part IV of the Constitution from Articles 36 to
51. The framers of the Constitution borrowed this idea from the Irish Constitution of 1937, which had
copied it from the Spanish Constitution. Dr B R Ambedkar described these principles as ‘novel features’
of the Indian Constitution.
Option 1 is correct: Article 42 of the Constitution of India states that the state shall make provisions for
ensuring just and humane conditions of work and maternity relief.
Option 2 is incorrect: Article 4 of the Universal Declaration of Human Rights (UDHR) states that no one
shall be held in slavery or servitude. Slavery and the slave trade shall be prohibited in all their forms.
Option 3 is correct: Article 48 of the Constitution of India provides that the State shall endeavour to
organise agriculture and animal husbandry on modern and scientific lines and shall, in particular, take
steps for preserving and improving the breeds, and prohibiting the slaughter, of cows and calves and
other milch and draught cattle.
Option 4 is correct: Article 51 of the Constitution of India states the promotion of international peace
and security and maintain just and honourable relations between nations; to foster respect for
international law and treaty obligations, and to encourage settlement of international disputes by
arbitration
Option 5 is incorrect: Article 17 of the Universal Declaration of Human Rights (UDHR) states that
everyone has the right to own property alone as well as in association with others. No one shall be
arbitrarily deprived of his property. Similar provisions are stated in article 300A of the constitution of
India which requires the state to follow due procedure and authority of law to deprive a person of his or
her private property. But it is not a part of the DPSPs.

Forum Learning Centre: Delhi - 2nd Floor, IAPL House, 19 Pusa Road, Karol Bagh, New Delhi - 110005 | Patna - 2nd floor, AG Palace, E Boring Canal
Road, Patna, Bihar 800001 | Hyderabad - 1st & 2nd Floor, SM Plaza, RTC X Rd, Indira Park Road, Jawahar Nagar, Hyderabad, Telangana 500020
9311740400, 9311740900 | https://academy.forumias.com | admissions@forumias.academy | helpdesk@forumias.academy
Page 9 of 47

PTS 2024 | B4 | L1 Test 1 - Solutions |

Since, statement 2 and 5 are not the part of DPSPs. Hence, correct answer is option a.
Source: Ch-8, DPSPs, Laxmikanth
https://www.un.org/en/about-us/universal-declaration-of-human-rights

Q.19)
Ans) d
Exp) Option d is the correct answer.
The Constitution has conferred a very extensive jurisdiction and vast powers on the Supreme Court. It is
not only a Federal Court like the American Supreme Court but also a final court of appeal like the British
House of Lords (the Upper House of the British Parliament). It is also the final interpreter and guardian
of the Constitution and guarantor of the fundamental rights of the citizens. Further, it has advisory and
supervisory powers.
Statement 1 is incorrect: Disputes relating to the election of members of Parliament and state
legislatures are heard by High Court (not Supreme court) under its original jurisdiction.
Statement 2 is incorrect: Any suit brought before the Supreme Court by a private citizen against the
Centre or a state cannot be entertained under original Jurisdiction of the Supreme Court.
Statement 3 is incorrect: Original jurisdiction of the Supreme Court does not extend to
a dispute arising out of any pre-Constitution treaty, agreement, covenant, engagement, sanad or other
similar instrument.
Source: C-26, Supreme Court, Laxmikanth

Q.20)
Ans) c
Exp) Option c is the correct answer.
The Covid Pandemic gave a boost to the online gaming market in India and people are now more willing
to pay to play. According to analysts, over 24 million Indians are added to the payment-linked gaming
market. Recently, Union government has proposed measures for regulating online gaming in a draft
amendment to the Information Technology (Intermediary Liability and Digital Media Ethics Code) Rules,
2021.
Statement 1 is correct: The regulation of the online gaming industry in India is a complex issue and has
been the subject of much debate in recent years. There are currently no specific laws or regulations in
place that govern the industry, and the industry is largely self- regulated. However, certain laws such as
the Information Technology Act, the Indian Contract Act, and the Public Gambling Act have been used to
regulate the industry.
Statement 2 is correct: In India, 100% foreign direct investment is allowed under the automatic route
for online gaming which falls under electronic system, IT and BPM sectors. This means that approval
from the appropriate ministries is not required for such investments.
Source:https://www.indiatimes.com/explainers/news/how-rapidly-is-the-gaming-industry-growing-
in-india-589059.html
https://www.investindia.gov.in/sector/media/gaming#:~:text=The%20gaming%20industry%20is%20at
,INR%20380%20bn%20by%202026.&text=India%27s%20percentage%20of%20new%20paying,)%20r
eaching%2050%25%20in%202021
https://indianexpress.com/article/opinion/columns/online-gaming-regulation-gst-8373955/
https://www.thehindu.com/opinion/editorial/gaming-and-gambling-the-hindu-editorial-on-the-
centres-move-to-regulate-online-gaming/article66338985.ece

Forum Learning Centre: Delhi - 2nd Floor, IAPL House, 19 Pusa Road, Karol Bagh, New Delhi - 110005 | Patna - 2nd floor, AG Palace, E Boring Canal
Road, Patna, Bihar 800001 | Hyderabad - 1st & 2nd Floor, SM Plaza, RTC X Rd, Indira Park Road, Jawahar Nagar, Hyderabad, Telangana 500020
9311740400, 9311740900 | https://academy.forumias.com | admissions@forumias.academy | helpdesk@forumias.academy
Page 10 of 47

PTS 2024 | B4 | L1 Test 1 - Solutions |

Q.21)
Ans) a
Exp) Option a is the correct answer.
Judicial review is the power of the judiciary to examine the constitutionality of legislative enactments and
executive orders of both the Central and State governments. On examination, if they are found to be
violative of the Constitution (ultra vires), they can be declared as illegal, unconstitutional and invalid (null
and void) by the judiciary.
Statement 1 is correct: According to the doctrine of the ‘basic structure’ (or ‘basic features’) of the
Constitution, the constituent power of Parliament under Article 368 does not enable it to alter the ‘basic
structure’ of the Constitution. And since the power of judicial review is a part of the basic structure of
the constitution, thus it cannot be curtailed by even by a constitutional amendment.
Statement 2 is correct: Although the Supreme Court is yet to define or clarify as to what constitutes the
‘basic structure’ of the Constitution, elements of the basic structure have emerged over a period of time
from various Supreme Court judgments. In I.R. Coelho case (2007), judicial review has emerged as a basic
structure of the constitution.
Knowledge Base:
Article 31B saves the acts and regulations included in the Ninth Schedule from being challenged and
invalidated on the ground of contravention of any of the Fundamental Rights. Article 31B along with the
Ninth Schedule was added by the 1st Constitutional Amendment Act of 1951.
In a significant judgement delivered in I.R. Coelho case (2007), the Supreme Court ruled that there could
not be any blanket immunity from judicial review of laws included in the Ninth Schedule. The court held
that judicial review is a ‘basic feature’ of the constitution and it could not be taken away by putting a
law under the Ninth Schedule. It said that the laws placed under the Ninth Schedule after April 24, 1973,
are open to challenge in court if they violated Fundamental Rights guaranteed under the Articles 14, 15, 19
and 21 or the ‘basic structure’ of the Constitution.
Source: Ch-27, Judicial review and chapter 11 Basic Structure of the Constitution, Laxmikanth

Q.22)
Ans) b
Exp) Option b is the correct answer.
Statement 1 is incorrect: The limit on loans taken by the Centre is decided by the Parliament, whereas
the limit on the loans taken by the state governments is decided by the respective state legislatures.
Statement 2 is correct: The loans given by the Centre to the states, or guarantees made by the Centre
with regards to a loan taken by the state from elsewhere is charged upon the Consolidated Fund of India.
Statement 3 is correct: A State with any outstanding (unpaid) loan - either taken from the Centre, or
regarding which the Centre had made guarantees, cannot take any further loans without the consent of
the Centre.
Knowledge Base:
1) The Centre may borrow both within as well as outside India.
2) State governments can only borrow within India.
3) The state governments take loans on the guarantee of the Consolidated Fund of the state, whereas the
Central Government takes loans on the security of the Consolidated Fund of India.
Source: Polity by Laxmikanth, 5th edition, Ch-14

Forum Learning Centre: Delhi - 2nd Floor, IAPL House, 19 Pusa Road, Karol Bagh, New Delhi - 110005 | Patna - 2nd floor, AG Palace, E Boring Canal
Road, Patna, Bihar 800001 | Hyderabad - 1st & 2nd Floor, SM Plaza, RTC X Rd, Indira Park Road, Jawahar Nagar, Hyderabad, Telangana 500020
9311740400, 9311740900 | https://academy.forumias.com | admissions@forumias.academy | helpdesk@forumias.academy
Page 11 of 47

PTS 2024 | B4 | L1 Test 1 - Solutions |

Q.23)
Ans) b
Exp) Option b is the correct answer.
The Citizenship Act, 1955 prescribes five ways of acquiring citizenship, viz, birth, descent, registration,
naturalisation and incorporation of territory.
statement 1 is incorrect. A person born in India on or after January 26, 1950 but before July 1, 1987 is a
citizen of India by birth irrespective of the nationality of his parents. Further, those born in India on or
after December 3, 2004 are considered citizens of India only if both of their parents are citizens of
India or one of whose parents is a citizen of India and the other is not an illegal migrant at the time of
their birth.
Statement 2 is incorrect. A person born outside India on or after December 10, 1992 is considered as a
citizen of India if either of his parents is a citizen of India at the time of his birth. December 3, 2004
onwards, a person born outside India shall not be a citizen of India by descent, unless his birth is
registered at an Indian consulate within one year of the date of birth or with the permission of the
Central Government, after the expiry of the said period.
statement 3 is correct. The Central Government may, on an application, register as a citizen of India any
person if he is a person of Indian origin who is ordinarily resident in India for seven years before
making an application for registration.
Statement 4 is correct: The Central Government may, on an application, grant a certificate of
naturalisation to any person if he during the fourteen years immediately preceding the said period of
twelve months, he has either resided in India or been in the service of a Government in India, or partly
the one and partly the other, for periods amounting in the aggregate to not less than eleven years.
Source: Indian Polity by Laxmikant – 6th Edition – Chapter 6 – Citizenship.

Q.24)
Ans) b
Exp) Option b is the correct answer.
The Government of India Act of 1935 marked one of the important milestones towards a completely
responsible government in India. It was a lengthy and detailed document having 321 Sections and 10
Schedules.
Some of the features of the act are as follows:
1) It abolished dyarchy in the provinces and introduced ‘provincial autonomy’ in its place. The
provinces were allowed to act as autonomous units of administration in their defined spheres.
Moreover, the Act introduced responsible governments in provinces, that is, the governor was
required to act with the advice of ministers responsible to the provincial legislature. This came into
effect in 1937 and was discontinued in 1939. Hence, Statement 1 is correct.
2) It provided for the adoption of dyarchy at the Centre. Consequently, the federal subjects were divided
into reserved subjects and transferred subjects. However, this provision of the Act did not come into
operation at all.
3) It introduced bicameralism in six out of eleven provinces. Thus, the legislatures of Bengal, Bombay,
Madras, Bihar, Assam and the United Provinces were made bicameral consisting of a legislative council
(upper house) and a legislative assembly (lower house). However, many restrictions were placed on
them.
4) It further extended the principle of communal representation by providing separate electorates for
depressed classes (scheduled castes), women and labour (workers).
5) It abolished the Council of India, established by the Government of India Act of 1858. The secretary of
state for India was provided with a team of advisors.

Forum Learning Centre: Delhi - 2nd Floor, IAPL House, 19 Pusa Road, Karol Bagh, New Delhi - 110005 | Patna - 2nd floor, AG Palace, E Boring Canal
Road, Patna, Bihar 800001 | Hyderabad - 1st & 2nd Floor, SM Plaza, RTC X Rd, Indira Park Road, Jawahar Nagar, Hyderabad, Telangana 500020
9311740400, 9311740900 | https://academy.forumias.com | admissions@forumias.academy | helpdesk@forumias.academy
Page 12 of 47

PTS 2024 | B4 | L1 Test 1 - Solutions |

6) It provided for the establishment of a Reserve Bank of India to control the currency and credit of the
country. Hence, Statement 3 is correct.
Whereas-
Statement 2 is incorrect: The Government of India Act of 1919 created a new office of the High
Commissioner for India in London and transferred to him some of the functions hitherto performed by
the Secretary of State for India.
Source: chapter 1 m laxmikant 5th edition

Q.25)
Ans) c
Exp) Option c is the correct answer.
The Union Commerce and industry minister has launched the mentorship, advisory, assistance, resilience
and growth (MAARG) platform on 16th January 2023 in New Delhi.
MAARG Platform is the National Mentorship Platform by Start-up India to help and guide new start-ups
to grow and flourish. It is a one-stop platform to facilitate mentorship for start-ups across diverse
sectors, functions, stages, geographies, and backgrounds.
It is launched by the Department for Promotion of Industry and Internal Trade (DPIIT) under the Ministry
of Commerce and Industry. The objectives are
1) To provide sector-focused guidance, handholding, and support to start-ups throughout their
lifecycle.
2) To establish a formalized and structured platform that facilitates intelligent matchmaking between the
mentors and their respective mentees
3) To facilitate efficient and expert mentorship for start-ups and build an outcome-oriented mechanism
that allows timely tracking of mentor-mentee engagements.
Source: https://economictimes.indiatimes.com/tech/startups/piyush-goyal-to-launch-maarg-
platform-to-aid-startup-entrepreneur-mentorship/articleshow/96971383.cms

Q.26)
Ans) d
Exp) Option d is the correct answer.
Writ jurisdiction refers to the authority or power of a higher court to issue writs, which are formal
written orders, in order to enforce or protect certain legal rights.
Statement 1 is incorrect: The writ jurisdiction of the Supreme Court with regards to fundamental
rights is not exclusive. Article 32 of the Indian Constitution gives the Supreme Court the power to issue
writs for the enforcement of fundamental rights, but High Courts also have this power under Article 226.
Statement 2 is incorrect: Article 32(2) of the Constitution provides that the Parliament may by law
empower the Supreme Court to issue writs for purposes other than the enforcement of Fundamental
Rights. The President doesn’t confer this power, but the Parliament may.
Source: Indian Polity by Laxmikanth, 6th Edition Chapter-26

Q.27)
Ans) c
Exp) Option c is the correct answer.
The Indian Constitution contain the rule of ‘immunity from mutual taxation’ and makes several provisions
in this regard.
Statement 1 is incorrect: The property of Centre is exempted from all taxes imposed by a state or any
authority within a state like municipalities, district boards, panchayats and so on. The corporations or

Forum Learning Centre: Delhi - 2nd Floor, IAPL House, 19 Pusa Road, Karol Bagh, New Delhi - 110005 | Patna - 2nd floor, AG Palace, E Boring Canal
Road, Patna, Bihar 800001 | Hyderabad - 1st & 2nd Floor, SM Plaza, RTC X Rd, Indira Park Road, Jawahar Nagar, Hyderabad, Telangana 500020
9311740400, 9311740900 | https://academy.forumias.com | admissions@forumias.academy | helpdesk@forumias.academy
Page 13 of 47

PTS 2024 | B4 | L1 Test 1 - Solutions |

the companies created by the Central government are not immune from state taxation or local
taxation. The reason is that a corporation or a company is a separate legal entity.
Statement 2 is correct: The property and income of local authorities situated within a state are not
exempted from the Central taxation. Similarly, the property or income of corporations and companies
owned by a state can be taxed by the Centre
Statement 3 is correct: The Centre has the exclusive power to levy customs duties, and this extends to
goods imported or exported by a state. The Supreme Court, in an advisory opinion (1963), held that the
immunity granted to a state in respect of Central taxation does not extend to the duties of customs or
duties of excise.
Source: Indian Polity by Laxmikanth, 6th Edition Chapter-14

Q.28)
Ans) c
Exp) Option c is the correct answer.
Article 22 grants protection to persons who are arrested or detained.
Statements 1 and 2 are correct: Article 22 provides protection in case of both punitive and preventive
detention. Article 22 has two parts–the first part deals with the cases of punitive detention under
ordinary law and the second part deals with the cases of preventive detention law.
The second part of Article 22 grants protection to persons who are arrested or detained under a
preventive detention law. This protection is available to both citizens as well as aliens.
Statement 3 is incorrect: Article 22 provides for that the detention of a person cannot exceed three
months unless advisory board reports sufficient cause for extended detention. The board is to consist of
judges of a high court. Thus, if the advisory board find the sufficient reason the preventive detention
can be extended for more than three months.
Statement 4 is correct: Article 22 authorizes the Parliament to prescribe:
(a) the circumstances and the classes of cases in which a person can be detained for more than three
months under a preventive detention law without obtaining the opinion of an advisory board
(b) the maximum period for which a person can be detained in any classes of cases under a preventive
detention law; and
(c) the procedure to be followed by an advisory board in an inquiry.
Knowledge Base: The Constitution has divided the legislative power with regard to preventive detention
between the Parliament and the state legislatures. The Parliament has exclusive authority to make a law
of preventive detention for reasons connected with defence, foreign affairs and the security of India.
Both the Parliament as well as the state legislatures can concurrently make a law of preventive
detention for reasons connected with the security of a state, the maintenance of public order and the
maintenance of supplies and services essential to the community.
Source: Indian Polity by Laxmikanth, 6th Edition Chapter-7

Q.29)
Ans) c
Exp) option c is the correct answer
The establishment of Family Courts was done with a view to promote conciliation and secure speedy
settlement of disputes relating to marriage and family affairs.
Statement 1 is correct: There are no specific provisions regarding Family Courts in the Indian
Constitution. The establishment of Family Courts is based on legislation i. e. Family Courts Act, 1984
which was enacted to provide for the establishment of Family Courts.

Forum Learning Centre: Delhi - 2nd Floor, IAPL House, 19 Pusa Road, Karol Bagh, New Delhi - 110005 | Patna - 2nd floor, AG Palace, E Boring Canal
Road, Patna, Bihar 800001 | Hyderabad - 1st & 2nd Floor, SM Plaza, RTC X Rd, Indira Park Road, Jawahar Nagar, Hyderabad, Telangana 500020
9311740400, 9311740900 | https://academy.forumias.com | admissions@forumias.academy | helpdesk@forumias.academy
Page 14 of 47

PTS 2024 | B4 | L1 Test 1 - Solutions |

Statement 2 is correct: Family Courts Act, 1984 makes it obligatory on the State Governments to set up a
Family Court in every city or town with a population exceeding one million.
Statement 3 is correct: The Family Court Act makes it obligatory on the part of the Family Court to
endeavour, in the first instance to effect a reconciliation or a settlement between the parties to a
family dispute. During this stage, the proceedings will be informal and rigid rules of procedure shall not
apply.
Source: Indian Polity by Laxmikanth, 6th Edition Chapter-36

Q.30)
Ans) b
Exp) Option b is the correct answer.
Recently, Supreme Court held that the public cannot be given free access to a charge sheet or a final
investigation report as it is not a public document.
Statement 1 is correct: An FIR is the first document that is filed when police are informed of a
cognizable offense. It sets the process of investigation in motion. On the other hand, the chargesheet is
the final report that is filed after the investigation is complete and the evidence has been collected.
Statement 2 is correct: The purpose of an FIR is to initiate the investigation into the commission of an
offense, while the purpose of a chargesheet is to provide evidence to support the allegations against the
accused and to prosecute the accused in a court of law.
Statement 3 is incorrect: FIR and Chargesheet cannot be withdrawn. There is no such provision to
withdraw FIR or chargesheet under The Code of Criminal Procedure, 1973. However, Delhi High Court has
observed that the FIR and chargesheet can be quashed if the allegations made in the FIR or complaint or
the evidence collected, though remaining uncontroverted, do not disclose the commission of an offence.
Statement 4 is incorrect: FIR is a public document prepared u/s-154 of Cr. P.C. A certified copy of an FIR
can be given in evidence. A copy of the FIR can be given to the accused only under the order of the court
after the court has taken cognizance of the case and not before. However, Supreme Court (SC) recently,
has ruled that chargesheets are not 'public documents' and enabling their free public access violates the
provisions of the Criminal Code of Procedure (CrPC) as it compromises the rights of the accused, victim,
and the investigation agencies.
Source) https://www.hindustantimes.com/cities/delhi-news/not-a-public-document-supreme-court-
refuses-to-make-charge-sheets-public-101674239751535.html
https://tripakshalitigation.com/difference-between-fir-and-charge-sheet/
https://www.legalserviceindia.com/legal/article-1338-what-is-fir-and-chargesheet-.html

Q.31)
Ans) b
Exp) Option b is the correct answer.
The Fundamental Rights have been enshrined in Part III of the Constitution of India from Articles 12 to 35.
Statement 1 is correct. The Right to Freedom is enumerated under Articles 19, 20, 21A and 22 of the of the
Constitution of India. Article 19 (a) (ii) guarantees every person the freedom of Assembly peacefully and
without arms.
Statement 2 is incorrect. The Freedom of Assembly comprises the right to hold public meetings,
processions and demonstrations. Only public land may be used to exercise this freedom, and the
gathering must be unarmed and peaceful. This provision does not protect assemblies that are violent,
disorderly, riotous, or that disturb the public peace or use arms. The right to strike is not a part of this
fundamental right.

Forum Learning Centre: Delhi - 2nd Floor, IAPL House, 19 Pusa Road, Karol Bagh, New Delhi - 110005 | Patna - 2nd floor, AG Palace, E Boring Canal
Road, Patna, Bihar 800001 | Hyderabad - 1st & 2nd Floor, SM Plaza, RTC X Rd, Indira Park Road, Jawahar Nagar, Hyderabad, Telangana 500020
9311740400, 9311740900 | https://academy.forumias.com | admissions@forumias.academy | helpdesk@forumias.academy
Page 15 of 47

PTS 2024 | B4 | L1 Test 1 - Solutions |

Statement 3 is correct. The State can impose reasonable restrictions on the exercise of right of assembly
on two grounds:
1) sovereignty and integrity of India, and
2) public order including the maintenance of traffic in the area concerned.
Source: M. Laxmikanth Indian Polity (Sixth Edition) Page 199

Q.32)
Ans) c
Exp) Option c is the correct answer.
Statement 1 is correct. Part XIII of the Constitution of India under Articles 301 to 307 talks about trade,
commerce and intercourse within the Indian territory. Article 301 states that trade, commerce and
intercourse throughout the territory of India shall be free. In order to promote the free flow of trade,
commerce, and intercourse in the country, the purpose of this provision is to remove the barriers that
currently exist at the borders between the states and to combine them as one unit.
Statement 2 is correct. Article 301 declares that trade, commerce and intercourse throughout the
territory of India shall be free. The freedom under this provision is not confined to interstate trade,
commerce and intercourse but also extends to intra-state trade, commerce and intercourse.
Statement 3 is correct. If it is in the public interest, Parliament may impose restrictions on the freedom
of trade, commerce and intercourse between states or within a state. Except in case of a shortage of
goods in any part of India, the Parliament is not allowed to favour one state over another or discriminate
between the states.
Source: M. Laxmikanth Indian Polity Chapter 15

Q.33)
Ans) c
Exp) Option c is the correct answer.
Federal Features of the Indian Constitution: There are a total of seven federal features of the
Constitution of India. These are:
1) Dual Polity
2) Written Constitution
3) Division of Powers
4) Supremacy of the Constitution
5) Rigid Constitution
6) Independent Judiciary
7) Bicameralism
Dual Polity: The Constitution of India provides a dual polity composed of the Union at the Centre and
the states at the periphery. Each has been provided with sovereign authority to exercise in the sphere
that the Constitution has assigned to them separately. The Union government oversees matters of
national importance like defence, international affairs, the currency, communication, and others. The
state governments deals with the issues related to public order, agriculture, health, local governance, and
other issues of regional and local importance.
Source: M. Laxmikanth Indian Polity Page-306

Q.34)
Ans) b
Exp) Option b is the correct answer.

Forum Learning Centre: Delhi - 2nd Floor, IAPL House, 19 Pusa Road, Karol Bagh, New Delhi - 110005 | Patna - 2nd floor, AG Palace, E Boring Canal
Road, Patna, Bihar 800001 | Hyderabad - 1st & 2nd Floor, SM Plaza, RTC X Rd, Indira Park Road, Jawahar Nagar, Hyderabad, Telangana 500020
9311740400, 9311740900 | https://academy.forumias.com | admissions@forumias.academy | helpdesk@forumias.academy
Page 16 of 47

PTS 2024 | B4 | L1 Test 1 - Solutions |


Article 262 of the Constitution provides for the adjudication of inter-state water disputes. It has two
provisions.
Statement 1 is correct. The Parliament of India may, by law, provide for the adjudication of any dispute
related to the use, distribution and control of waters of any inter-state river and river valley.
Statement 2 is incorrect. Parliament can additionally state that no other court, including the Supreme
Court of India, has any jurisdiction over any such dispute or complaint. The River Boards Act (1956) and
the Inter-State Water Disputes Act (1956) are two laws that the Parliament enacted in accordance with
this provision.
Statement 3 is correct. A dispute over the waters of an inter-state river or river valley between two or
more states may be adjudicated by an ad-hoc tribunal established by the Central government under the
Inter-State Water Disputes Act of 1956. The tribunal's decision would be final and binding on the
disputing parties. Any water dispute that may be referred to such a tribunal under this Act is outside the
purview of the Supreme Court and all other courts.
Source: M. Laxmikanth Indian Polity Chapter-15

Q.35)
Ans) c
Exp) Option c is the correct answer.
Recently, Food and Agriculture Organization (FAO) of the United Nations launched report on Urban
forestry and urban greening in drylands under the framework of FAO’s Green Urban Oases Programme.
Green Urban Oases (GUO) Programme was launched in 2021 by FAO. It contributes to the FAO Green
Cities initiative, which was launched in 2020. It aims to improve the resilience of dryland cities by tackling
climate, health, food and economic challenges.
The objectives of the program are:
1) To transform dryland cities into ‘green urban oases’ by strengthening their overall resilience to
climatic, health, food and economic crises for the improved health and well-being of urban
communities.
2) It focuses on developing policy, technical capacity and outlines several pathways for transforming
urban spaces by planting trees.
Source: https://www.fao.org/documents/card/en/c/cb5783en/

Q.36)
Ans) c
Exp) Option c is the correct answer.
Statement 1 is correct. A law so enacted applies only to those states which have passed the resolutions.
However, any other state may adopt it afterwards by passing a resolution to that effect in its legislature.
Statement 2 is incorrect. The bill to give effect to such a law can be initiated in any house of the
Parliament.
Statement 3 is correct. Such a law can be amended or repealed only by the Parliament and not by the
legislatures of the concerned states. The effect of passing a resolution under the above provision is that
the Parliament becomes entitled to legislate with respect to a matter for which it has no power to make a
law. On the other hand, the state legislature ceases to have the power to make a law with respect to that
matter.
Statement 4 is correct. Wildlife (Protection) Act of 1972, Prize Competition Act, 1955, Water (Prevention
and Control of Pollution) Act, 1974, Urban Land (Ceiling and Regulation) Act, 1976, Transplantation of
Human Organs Act, 1994 were passed under this provision.
Source: Indian Polity by M. Laxmikant

Forum Learning Centre: Delhi - 2nd Floor, IAPL House, 19 Pusa Road, Karol Bagh, New Delhi - 110005 | Patna - 2nd floor, AG Palace, E Boring Canal
Road, Patna, Bihar 800001 | Hyderabad - 1st & 2nd Floor, SM Plaza, RTC X Rd, Indira Park Road, Jawahar Nagar, Hyderabad, Telangana 500020
9311740400, 9311740900 | https://academy.forumias.com | admissions@forumias.academy | helpdesk@forumias.academy
Page 17 of 47

PTS 2024 | B4 | L1 Test 1 - Solutions |

Q.37)
Ans) d
Exp) Option d is the correct answer.
Option d is correct. Under the Constitution, the jurisdiction of each state is confined to its own territory.
Hence, it is possible that the acts and records of one state may not be recognised in another state. To
remove any such difficulty, the Constitution contains the “Full Faith and Credit” clause which lays down
the following:
1) Full faith and credit is to be given throughout the territory of India to public acts, records and
judicial proceedings of the Centre and every state. The expression ‘public acts’ includes both
legislative and executive acts of the government.
2) The manner in which and the conditions under which such acts, records and proceedings are to be
proved and their effect determined would be as provided by the laws of Parliament.
3) Final judgements and orders of civil courts in any part of India are capable of execution anywhere
within India (without the necessity of a fresh suit upon the judgement). The rule applies only to civil
judgements and not to criminal judgement.
Source: Laxmikanth, Ch- 14, Centre–State Relations

Q.38)
Ans) a
Exp) Option a is the correct answer.
Article 270 deals with taxes that are levied and collected by the central government and how they are
distributed between the central government and the state government according to a predetermined
formula which is provided by the Finance Commission once every 5 years.
Option 1 is correct: The income-tax is levied and collected by the Centre but its proceeds are
distributed between the Centre and the states.
Option 2 is incorrect: Corporation Tax (Corporate tax) comes under Taxes Levied, Collected and
Retained by the Centre.
Option 3 is incorrect: Taxes on the sale of petroleum crude comes under Taxes Levied and Collected and
Retained by the States.
Option 4 is incorrect: The Central Government can't impose or levy tax on agricultural income. The
exemption clause is mentioned under Section 10 (1) of the Income Tax Act of India.
However, the state government can levy tax on agricultural income if the amount exceeds Rs. 5,000 per
year.
Source: https://fincomindia.nic.in/ShowContent.aspx?uid1=2&uid2=1&uid3=0&uid4=0

Q.39)
Ans) d
Exp) Option d is the correct answer.
Article 18 abolishes titles and makes certain provisions in that regard such as it prohibits the state from
conferring any title (except a military or academic distinction) on any body, whether a citizen or a
foreigner. It prohibits a citizen of India from accepting any title from any foreign state. A foreigner
holding any office of profit or trust under the state cannot accept any title from any foreign state without
the consent of the president.
Statement 1 is incorrect: In 1996, the Supreme Court upheld the constitutional validity of the National
Awards–Bharat Ratna, Padma Vibhushan, Padma Bhushan and Padma Sri. It ruled that these awards
should not be used as suffixes or prefixes to the names of awardees. Otherwise, they should forfeit the
awards.

Forum Learning Centre: Delhi - 2nd Floor, IAPL House, 19 Pusa Road, Karol Bagh, New Delhi - 110005 | Patna - 2nd floor, AG Palace, E Boring Canal
Road, Patna, Bihar 800001 | Hyderabad - 1st & 2nd Floor, SM Plaza, RTC X Rd, Indira Park Road, Jawahar Nagar, Hyderabad, Telangana 500020
9311740400, 9311740900 | https://academy.forumias.com | admissions@forumias.academy | helpdesk@forumias.academy
Page 18 of 47

PTS 2024 | B4 | L1 Test 1 - Solutions |

Statement 2 is correct: Supreme Court also upheld that these awards do not amount to ‘titles’ within the
meaning of Article 18 that prohibits only hereditary titles of nobility. Therefore, they are not violative of
Article 18 as the theory of equality does not mandate that merit should not be recognised.
Source: Laxmikanth, Ch-7: Fundamental Rights

Q.40)
Ans) b
Exp) Option b is the correct answer.
Monument Mitra scheme that entails adopting a heritage site and maintaining it, will be revamped for
private firms to partner for the upkeep of 1,000 ASI monuments.
Statement 1 is incorrect: Monument Mitra scheme was launched under the Ministry of Tourism earlier
and then transferred to the Ministry of Culture. At present, Ministry of Culture is the nodal ministry to
manage and coordinates the scheme.
Statement 2 is correct: The objective of the scheme is to ensure quality & inclusive provision of
amenities and facilities across heritage, natural, & tourist sites through the active participation of
private and public sector organizations and individuals.
Statement 3 is correct: Monument Mitra Scheme applies to all centrally protected heritage properties.
The government has set the target to hand over 500 sites under the revamped Monument Mitra Scheme
by the end of Azadi ka Amrit Mahotsav on 15 August 2023.
Statement 4 is incorrect: Corporate entities including both private and public units will adopt these
monuments as part of their Corporate Social Responsibility. These organizations would be known as
“Monument Mitras” for their collaboration initiative.
Source: https://www.thehindu.com/news/national/monument-mitra-scheme-to-be-
revamped/article66432240.ece
https://www.newindianexpress.com/nation/2023/jan/26/revised-version-of-monument-mitra-
scheme-to-be-launched-by-culture-min-next-month-2541577.html

Q.41)
Ans) b
Exp) Option b is the correct answer.
Zonal Councils are bodies that aim at fostering cooperation and coordination among States, Union
Territories, and the Centre. They function solely as deliberative and advisory bodies.
Statement 1 is incorrect- The Zonal Councils are statutory bodies, not constitutional ones, established
through the States Reorganisation Act of 1956. This act divided the country into five zones (Northern,
Central, Eastern, Western, and Southern) and assigned a zonal council for each zone.
Zonal councils have the objective of fostering cooperation and coordination among states, union
territories, and the Centre. Their discussions and recommendations cover various areas such as
economic and social planning, linguistic minorities, border disputes, interstate transport, and more. It's
important to note that they serve as deliberative and advisory bodies exclusively.
Statement 2 is correct- The home minister of the Central government serves as the chairman of all five
zonal councils. Each chief minister takes turns acting as the vice-chairman of the council, with a term of
one year at a time.
Statement 3 is correct- The following persons can be associated with the zonal council as advisors (i.e.,
without the right to vote in the meetings):
(1) a person nominated by the Planning Commission;
(2) chief secretary of the government of each state in the zone; and
(3) development commissioner of each state in the zone.
Source: Laxmikanth 6th edition, chapter 15

Forum Learning Centre: Delhi - 2nd Floor, IAPL House, 19 Pusa Road, Karol Bagh, New Delhi - 110005 | Patna - 2nd floor, AG Palace, E Boring Canal
Road, Patna, Bihar 800001 | Hyderabad - 1st & 2nd Floor, SM Plaza, RTC X Rd, Indira Park Road, Jawahar Nagar, Hyderabad, Telangana 500020
9311740400, 9311740900 | https://academy.forumias.com | admissions@forumias.academy | helpdesk@forumias.academy
Page 19 of 47

PTS 2024 | B4 | L1 Test 1 - Solutions |

Q.42)
Ans) a
Exp) Option a is the correct answer.
Martial law in India refers to the temporary imposition of military control during a state of emergency. It
involves the military assuming authority over civilian governance to maintain law and order.
Statement 1 is incorrect- Martial Laws affect Fundamental Rights only. On the other hand, National
Emergency affects not only Fundamental Rights but also Centre-state relations, distribution of revenues
and legislative powers between centre and states and may extend the tenure of the Parliament.
Statement 2 is incorrect- Martial Law is imposed in some specific areas of the country. On the other
hand, National Emergency is imposed either in the whole country or in any part of it.
Statement 3 is correct- Martial Law suspends the government and ordinary law courts. On the other
hand, National Emergency continues the government and ordinary law courts. During a National
Emergency in India, the government may exercise extraordinary powers to address the crisis. This
includes the suspension of certain fundamental rights, the centralization of authority, and the
establishment of special courts or tribunals to handle emergency-related matters. However, the ordinary
law courts continue to function, albeit with limitations on their jurisdiction during the emergency period.
Source: Lamikanth 6th edition, Chapter-7

Q.43)
Ans) b
Exp) Option b is the correct answer.
The preamble is a brief introductory statement that sets out the guiding purpose, principles, and
philosophy of the constitution. The Preamble to the Indian Constitution is based on Jawaharlal Nehru’s
“Objective Resolution,” which was endorsed by the Constitution’s framers.
Statement 1 is correct: Preamble is an integral part of the constitution, but it is neither a source of
power to the legislature nor a prohibition upon the powers of the legislature. The Preamble is a
statement of the ideals and principles that the Constitution.
Statement 2 is incorrect: Parliament can amend only that part of the Preamble, which is not the part of
basic structure of the Constitution.
Supreme Court held, in the historic case of Kesavananda Bharati (1973), that the Preamble is a part of the
Constitution. The Court stated that the opinion tendered by it in the Berubari Union (1960) in this regard
was wrong, and held that the Preamble can be amended, subject to the condition that no amendment is
done to the ‘basic features’. In other words, the Court held that the basic elements or the fundamental
features of the Constitution as contained in the Preamble cannot be altered by an amendment under
Article 368. The Preamble has been amended only once so far, in 1976, by the 42 nd Constitutional
Amendment Act. This amendment has added three new words- Socialist, Secular and Integrity - to the
Preamble.
Statement 3 is correct: Preamble is non-justiciable, that is, its provisions are not enforceable in courts
of law. Although the Preamble is not enforceable in court, it describes the Constitution’s goals and serves
as a guide for interpreting Articles when the language is unclear.
Source: Laxmikant, Chapter 4: Preamble

Q.44)
Ans) b
Exp) Option b is the correct answer.

Forum Learning Centre: Delhi - 2nd Floor, IAPL House, 19 Pusa Road, Karol Bagh, New Delhi - 110005 | Patna - 2nd floor, AG Palace, E Boring Canal
Road, Patna, Bihar 800001 | Hyderabad - 1st & 2nd Floor, SM Plaza, RTC X Rd, Indira Park Road, Jawahar Nagar, Hyderabad, Telangana 500020
9311740400, 9311740900 | https://academy.forumias.com | admissions@forumias.academy | helpdesk@forumias.academy
Page 20 of 47

PTS 2024 | B4 | L1 Test 1 - Solutions |

The Constitution of India, being federal in structure, divides all powers (legislative, executive and
financial) between the Centre and the states. Indian Constitution also divides the legislative powers
between the Centre and the states with respect to both the territory and the subjects of legislation.
Statement 1 is incorrect: The Constitution defines the territorial limits of the legislative powers vested in
the Centre and the states. A state legislature can make laws for the whole or any part of the state. The
laws made by a state legislature are not applicable outside the state, except when there is a sufficient
nexus between the state and the object.
Statement 2 is correct: The Parliament alone can make ‘extra-territorial legislation’. Thus, the laws of
the Parliament are also applicable to the Indian citizens and their property in any part of the world.
Source: Laxmikanth Chapter 14 Centre-State Relations

Q.45)
Ans) b
Exp) Option b is the correct answer.
Recently, a traditional art Ottan thullal of Kerala which has a history of over 300 years was performed in
Kerala School Kalolsavam 2023.
Statement 1 is correct: Thullal is a recite-and-dance art form of Kerala. The distinguishing factor of
Ottan thullal is the performer (dancer) himself singing (reciting) and playing the story, which is a
tedious task. Another person will recite the same verses. The accompanying instruments for Ottan thullal
are Mridangam and Idakka.
Statement 2 is incorrect: Ottan thullal was introduced in the 18th century by the famous Malayalam
poet Kunchan Nambiar (1705 - 1770). Kathakali was revived in the 1930s by the famous Malayali poet V.
N. Menon under the patronage of Mukunda Raja.
Statement 3 is correct: Thullal is famous for its humor and social satire and is marked by its simplicity.
Kunchan Nambiar used it as a medium to protest against the prevalent socio-political structure and
prejudices of the society in his time. It went on to become the most popular folk art presented in Kerala
temples.
Source: https://www.thehindu.com/news/national/kerala/kerala-school-kalolsavam-2023-the-need-
to-take-ottanthullal-forward/article66343344.ece

Q.46)
Ans) b
Exp) Option b is the correct answer.
The Seventh Schedule of the Constitution of India lists subjects that are divided into three lists: the
Union List, the State List, and the Concurrent List. The Union List consists of subjects on which only the
central government can make laws, the State List consists of subjects on which only the state
governments can make laws, and the Concurrent List consists of subjects on which both the central and
state governments can make laws.
Option 1 is correct: Prisons, reformatories, and other institutions of a like nature is included in the State
List under Entry 4. The management and administration of prisons and similar institutions are primarily
the responsibility of the state governments. Each state has its own laws and regulations regarding the
operation of prisons within its jurisdiction.
Option 2 is incorrect: Marriage and divorce subject is included in the Concurrent List under Entry 5.
Marriage and divorce laws are governed by personal laws based on religion, customary practices, and
specific communities. While there are central laws that apply to inter-religious marriages and divorce,
states also have the authority to enact laws and regulations related to marriage and divorce within their
jurisdictions.

Forum Learning Centre: Delhi - 2nd Floor, IAPL House, 19 Pusa Road, Karol Bagh, New Delhi - 110005 | Patna - 2nd floor, AG Palace, E Boring Canal
Road, Patna, Bihar 800001 | Hyderabad - 1st & 2nd Floor, SM Plaza, RTC X Rd, Indira Park Road, Jawahar Nagar, Hyderabad, Telangana 500020
9311740400, 9311740900 | https://academy.forumias.com | admissions@forumias.academy | helpdesk@forumias.academy
Page 21 of 47

PTS 2024 | B4 | L1 Test 1 - Solutions |

Option 3 is correct: According to Entry 14 of the State List, "Agriculture, including agricultural
education and research, protection against pests and prevention of plant diseases" falls within the
purview of the state governments. This means that the state governments have the exclusive authority
to legislate and make policies regarding agricultural activities, agricultural education and research, pest
control, and plant disease prevention within their respective states.
Option 4 is incorrect: Preventive detention subject is included in the Concurrent List under Entry 9.
Preventive detention refers to the detention of individuals without trial or conviction, based on the
anticipation of a potential threat to public safety, maintenance of public order, or national security. The
central and state governments have the authority to enact laws related to preventive detention.
Source: Laxmikanth
https://www.mea.gov.in/Images/pdf1/S7.pdf

Q.47)
Ans) a
Exp) Option a is the correct answer.
No one may be prosecuted and punished for the same offence more than once, according to Article 20(2).
This is known as the Double Jeopardy Doctrine.
Statement 1 is correct: The principle of double jeopardy is enshrined in Article 20(2) of the Constitution
of India, which states that no person can be prosecuted and punished for the same offense more than
once. This principle provides protection against being tried or punished multiple times for the same
offense, safeguarding individuals from harassment and ensuring fairness in the criminal justice system.
Statement 2 is incorrect: The protection against double jeopardy is available only in proceedings before a
court of law or a judicial tribunal. In other words, it is not available in proceedings before departmental
or administrative authorities as they are not of judicial nature.
In the case of Maqbool Hussain v. State of Bombay, a person arrived at an Indian airport from abroad. He
was found in possession of gold which was against the law at the time. Action was taken against him by
the customs authorities and the gold was confiscated. Later he was prosecuted before a criminal court
under the Foreign Exchange Regulation Act. The question was whether the plea of protection against
double jeopardy could be raised under Art. 20(2). The Supreme Court came to the conclusion that the
proceedings before the customs authorities did not constitute 'prosecution' of the appellant, and the
penalty imposed on him did not constitute 'punishment' by a judicial tribunal.
Source: Laxmikanth
https://legalserviceindia.com/legal/article-7909-rights-of-the-accused.html
https://blog.ipleaders.in/the-doctrine-of-double-jeopardy/#Ex_post_facto_law_Article_201

Q.48)
Ans) c
Exp) Option c is the correct answer.
Fabian socialism derives its name from the Fabian Society, a British socialist organization founded in the
late 19th century. Fabian socialists advocate for a gradual and evolutionary approach to socialism,
emphasizing the use of democratic processes and reform rather than revolution.
1) They believe in achieving socialist goals through incremental changes within existing democratic
structures.
2) Fabian socialists support the idea of a welfare state, progressive taxation, and government
intervention in the economy to promote social justice and equality.
3) It feels the necessity of ‘State’ as an ultimate arbiter of all disputes. Though power shall be rested in
the hands of the state, there shall be no opportunity to exercise it arbitrarily.

Forum Learning Centre: Delhi - 2nd Floor, IAPL House, 19 Pusa Road, Karol Bagh, New Delhi - 110005 | Patna - 2nd floor, AG Palace, E Boring Canal
Road, Patna, Bihar 800001 | Hyderabad - 1st & 2nd Floor, SM Plaza, RTC X Rd, Indira Park Road, Jawahar Nagar, Hyderabad, Telangana 500020
9311740400, 9311740900 | https://academy.forumias.com | admissions@forumias.academy | helpdesk@forumias.academy
Page 22 of 47

PTS 2024 | B4 | L1 Test 1 - Solutions |

4) It advocates nationalization of key industries in the economic structure of the society but did not
support management of entire economy by state.
Source: https://www.oxfordreference.com/view/10.1093/oi/authority.20110803095807198
https://www.britannica.com/event/Fabianism

Q.49)
Ans) d
Exp) Option d is the correct answer.
In India, the power of judicial review is derived from the Constitution, primarily from Articles 13, 32, 131,
and 226. The judiciary, including the High Courts and the Supreme Court, has the responsibility to
interpret the Constitution and ensure that all actions of the government, including legislative and
executive actions, are in conformity with its provisions.
Statement 1 is incorrect: High Courts in India have the power of judicial review over both state laws and
central laws. The power of judicial review allows the High Courts to examine the constitutional validity of
laws enacted by both the central government and state governments and strike them down if they are
found to be unconstitutional or in violation of the provisions of the Constitution.
Statement 2 is incorrect: The Supreme Court of India has the power of judicial review over state laws
and central laws. The Supreme Court can review state laws if they are found to be unconstitutional or
in contravention of the provisions of the Constitution. The constitutional validity of central laws is not a
prerequisite for the Supreme Court to review state laws. The Supreme Court can review and strike down
state laws if they violate fundamental rights or other constitutional provisions.
Article 32A was inserted into the constitution by the 42nd Amendment in 1976. Article 32A barred
reviewing of State laws unless constitutional validity of Central laws was also an issue. Subsequently, the
43rd amendment repealed Article 32A after the end of the emergency.
Source: Laxmikanth Ch- 26, Supreme Court
https://blog.ipleaders.in/significance-of-article-32-of-the-constitution-of-india-right-to-
constitutional-remedies/

Q.50)
Ans) b
Exp) Option b is the correct answer.
Bonds are debt instruments in which investors lend money to a company or a government for a set period
& receive interest payments in return.
Statement 1 is correct and statement 2 is incorrect: AT1 bonds are unsecured bonds that have
perpetual tenor. In other words, these bonds, issued by banks, have no maturity date. They have a call
option, which can be used by the banks to buy these bonds back from investors. But banks are not obliged
to use this call option and can opt to pay only interest on these bonds for eternity.
Statement 3 is correct: AT1 bonds were introduced by the Basel accord after the global financial crisis
of 2008 to protect depositors. These bonds are typically used by banks to bolster their core or tier-1
capital to meet the Basel-III norms. The Reserve Bank of India (RBI) is the regulator of AT-1 bonds.
If the RBI feels that a bank is tottering on the brink and needs a rescue, it can simply ask the bank to
write-off its outstanding AT-1 bonds without consulting its investors.
Source: https://indianexpress.com/article/explained/explained-economics/yes-bank-at1-bonds-
bombay-high-court-8395311/
https://www.outlookindia.com/business/explainer-what-are-at1-bonds-what-investors-must-know-
about-bombay-hc-order-on-yes-bank-at1-bond-case-news-255539

Forum Learning Centre: Delhi - 2nd Floor, IAPL House, 19 Pusa Road, Karol Bagh, New Delhi - 110005 | Patna - 2nd floor, AG Palace, E Boring Canal
Road, Patna, Bihar 800001 | Hyderabad - 1st & 2nd Floor, SM Plaza, RTC X Rd, Indira Park Road, Jawahar Nagar, Hyderabad, Telangana 500020
9311740400, 9311740900 | https://academy.forumias.com | admissions@forumias.academy | helpdesk@forumias.academy
Page 23 of 47

PTS 2024 | B4 | L1 Test 1 - Solutions |

Q.51)
Ans) b
Exp) Option b is the correct answer.
The Advisory jurisdiction of the Supreme Court in Constitution of India is sourced from the Government
of India Act, 1935 wherein the provision of Section 213(1) of the Government of India Act, 1935 conferred
an advisory function upon the Federal Court.
Statement 1 is s correct: Constitution (Article 143) authorises the President to seek the opinion of the
Supreme Court in the two categories of matters.
1) On any question of law or fact of public importance which has arisen or which is likely to arise.
2) On any dispute arising out of any pre-constitution treaty, agreement, covenant, engagement, sanad
or other similar instruments.
In the first case, the Supreme Court may tender or may refuse to tender its opinion to the president. But,
in the second case, the Supreme Court ‘must’ tender its opinion to the President. Thus, the Supreme
Court must tender its opinion to the president in case of disputes over pre-constitution treaty.
Statement 2 is incorrect: Advice given under Article 143 is not adjudication but consultation. It is an
advisory function designed to assist the President (the Executive). There is to be no judgement, decree or
order but opinion to be forwarded to the President in a report. The views taken by the Court is not
binding on the President and it is not law within Article 141.
Statement 3 is correct: A Constitution Bench is a bench of the Supreme Court having five or more
judges on it. Constitution Benches are exceptions, set up only if one or more of the following
circumstances exist:
1) The case involves a substantial question of law pertaining to the interpretation of the Constitution
[see Article 145(3) of the Constitution, which mandates that such matters be heard by a bench of not
less than five judges].
2) The President of India has sought the Supreme Court’s opinion on a question of fact or law under
Article 143 of the Constitution.
3) Different benches of the Supreme Court have delivered conflicting judgments on the same point of
law, thus warranting a definitive pronouncement by a larger bench.
Source: Laxmikanth, Ch-26: Supreme Court
https://www.legalserviceindia.com/legal/article-576-advisory-jurisdiction-of-the-supreme-court-is-
the-court-bound-to-give-its-opinion.html
https://archive.pib.gov.in/archive/releases98/lyr2002/raug2002/21082002/r210820021.html
https://www.thehindu.com/news/national/explained-the-constitution-bench-of-the-supreme-
court/article65955010.ece

Q.52)
Ans) a
Exp) Option a is the correct answer.
The Preamble to the Indian Constitution is based on the ‘Objectives Resolution’, drafted and moved by
Pandit Nehru, and adopted by the Constituent Assembly. The Preamble of India reflects the philosophy
and guiding principles of the Constitution. It outlines the aspirations and objectives of the framers of the
Constitution, providing a vision for the nation.
Option 1 is correct: The Preamble of India assures the citizens the ideal of equality of opportunity. It
emphasizes the commitment to securing for all citizens the equality of status and opportunity.
Option 2 is correct: The Preamble guarantees the citizens the ideal of liberty of thought, expression,
belief, faith, and worship. It recognizes and protects the freedom of individuals to have their own
opinions, express themselves, and follow their chosen beliefs and faiths.

Forum Learning Centre: Delhi - 2nd Floor, IAPL House, 19 Pusa Road, Karol Bagh, New Delhi - 110005 | Patna - 2nd floor, AG Palace, E Boring Canal
Road, Patna, Bihar 800001 | Hyderabad - 1st & 2nd Floor, SM Plaza, RTC X Rd, Indira Park Road, Jawahar Nagar, Hyderabad, Telangana 500020
9311740400, 9311740900 | https://academy.forumias.com | admissions@forumias.academy | helpdesk@forumias.academy
Page 24 of 47

PTS 2024 | B4 | L1 Test 1 - Solutions |

Option 3 is incorrect: Preservation of culture is not explicitly mentioned in the Preamble. While the
Constitution of India recognizes the importance of preserving and promoting diverse cultural traditions
and heritage, the specific phrase "preservation of culture" is not mentioned in the Preamble.
Option 4 is incorrect: Right to privacy is not explicitly mentioned in the Preamble. However, the
Supreme Court of India has recognized the right to privacy as a fundamental right derived from the
right to life and personal liberty guaranteed under Article 21 of the Constitution.
Option 5 is incorrect: Economic prosperity is not explicitly mentioned in the Preamble. While the
Constitution of India aims to promote economic development and the welfare of its citizens, the specific
phrase "economic prosperity" is not mentioned in the Preamble.
Source: Ch-3, DD Basu, Philosophy of the Constitution

Q.53)
Ans) c
Exp) Option c is the correct answer.
The Minerva Mills case is a landmark judgment delivered by the Supreme Court of India in 1980. The case
primarily dealt with the conflict between the Fundamental Rights (Part III) and the Directive Principles of
State Policy (Part IV) of the Indian Constitution.
In the Minerva Mills case (1980), the Supreme Court held that ‘the Indian Constitution is founded on the
bedrock of the balance between the Fundamental Rights and the Directive Principles. They together
constitute the core of commitment to social revolution. They are like two wheels of a chariot, one no
less than the other. To give absolute primacy to one over the other is to disturb the harmony of the
Constitution. This harmony and balance between the two is an essential feature of the basic structure of
the Constitution.
Option a is incorrect: The Maneka Gandhi case is a significant case related to the right to personal
liberty and the due process of law. In this case, the Supreme Court expanded the interpretation of Article
21 (right to life and personal liberty) to include a wider range of rights.
Option b is incorrect: The D C Wadhwa case held that the legislative power of the executive to
promulgate ordinances is to be used in exceptional circumstances and not as a substitute for the law-
making power of the legislature.
Option d is incorrect: The Kesavananda Bharati case is one of the most significant cases in Indian
constitutional history. The Supreme Court, in this case, laid down the doctrine of "basic structure"
which restricts the amending power of the Parliament.
Source: Laxmikanth-DPSP

Q.54)
Ans) b
Exp) Option b is the correct answer.
Judicial restraint refers to the approach or philosophy adopted by judges and courts in which they
exercise self-restraint and limit their own authority to interfere with the decisions and actions of the
other branches of government, such as the legislature and the executive.
It is based on the understanding by emphasizing adherence to the original intent of the Constitution's
framers as a guiding principle for judges practicing judicial restraint.
Option a is incorrect: Judicial restraint does not imply an absolute and unconditional deference to the
decisions of the political branches of government. While judicial restraint advocates for self-restraint and
a cautious approach by judges, it does not mean they must always defer to political decisions.
Option c is incorrect: The concept of judicial restraint is actually in contrast to judicial activism.
Judicial restraint calls for judges to exercise self-restraint and limit their involvement in policymaking or

Forum Learning Centre: Delhi - 2nd Floor, IAPL House, 19 Pusa Road, Karol Bagh, New Delhi - 110005 | Patna - 2nd floor, AG Palace, E Boring Canal
Road, Patna, Bihar 800001 | Hyderabad - 1st & 2nd Floor, SM Plaza, RTC X Rd, Indira Park Road, Jawahar Nagar, Hyderabad, Telangana 500020
9311740400, 9311740900 | https://academy.forumias.com | admissions@forumias.academy | helpdesk@forumias.academy
Page 25 of 47

PTS 2024 | B4 | L1 Test 1 - Solutions |


interpreting the law in a broad or expansive manner. On the other hand, judicial activism refers to a more
active role taken by judges in shaping public policy and interpreting the law expansively to achieve
certain social or political objectives.
Option d is incorrect: Judicial restraint does not imply a rigid or inflexible approach that limits the
judiciary's ability to interpret and apply the law. Rather, it emphasizes a cautious and limited exercise of
judicial power, with an understanding that judges should respect the roles of the other branches of
government and avoid overstepping their constitutional boundaries.
Source: Laxmikanth C-28
https://blog.ipleaders.in/judicial-
restraint/#:~:text=Judicial%20restraint%20is%20a%20theory,executive%20organs%20of%20the%20gov
ernment.
https://www.legalservicesindia.com/article/2019/Judicial-Activism-and-Judicial-Restraint.html

Q.55)
Ans) c
Exp) Option c is the correct answer.
Shri C V Raju from Etikoppaka village, Visakhapatnam, Andhra Pradesh has been conferred with Padma
Shri for preserving the traditional method of making Etikoppaka toys.
Option a is correct: Etikoppaka toys are traditional toys made by artisans of Etikoppaka village located
on the banks of Varaha River in Visakhapatnam district of Andhra Pradesh.
Option b is correct: Etikoppaka toys are made out of wood and are colored with natural dyes derived
from seeds, lacquer, bark, roots and leaves. The artisans mainly use the wood from trees known as
'ankudu' (Wrightia Tinctoria) that is soft in nature.
Option c is incorrect: Etikoppaka toys have no sharp edges. They are rounded on all sides and hence
present little chance of injury to children.
Option d is correct: Etikoppaka Toys have obtained their GI tag under Handicrafts category in the state
of Andhra Pradesh in 2017.
Source: https://www.thehindu.com/news/national/andhra-pradesh/padma-award-is-an-honour-for-
the-etikoppaka-toy-craft-says-cv-raju/article66436816.ece
https://vikaspedia.in/aspirational-districts/andhra-pradesh/vishakhapatnam/know-your-
district/etikoppaka-toys

Q.56)
Ans) a
Exp) Option a is the correct answer.
First Information Report (FIR): It is with the registration of an FIR that the police can begin their
investigations into a crime. The FIR usually mentions the date, time and place of the offence, details the
basic facts of the offence, including a description of the events. There is a prescribed form in which the
police registers an FIR and it is signed by the complainant. The complainant also has a legal right to get a
free copy of the FIR from the police.
Statements 1 is incorrect: The term First Information Report (FIR) is not defined in the Indian Penal
Code (IPC), Code of Criminal Procedure (CrPC), 1973, or in any other law. FIR is the information given to
a police officer as per the provisions of Section 154 of the Code of Criminal Procedure (CrPC).
There are three important elements of an FIR:
1) the information must relate to the commission of a cognizable offence
2) it should be given in writing or orally to the head of the police station and,

Forum Learning Centre: Delhi - 2nd Floor, IAPL House, 19 Pusa Road, Karol Bagh, New Delhi - 110005 | Patna - 2nd floor, AG Palace, E Boring Canal
Road, Patna, Bihar 800001 | Hyderabad - 1st & 2nd Floor, SM Plaza, RTC X Rd, Indira Park Road, Jawahar Nagar, Hyderabad, Telangana 500020
9311740400, 9311740900 | https://academy.forumias.com | admissions@forumias.academy | helpdesk@forumias.academy
Page 26 of 47

PTS 2024 | B4 | L1 Test 1 - Solutions |


3) it must be written down and signed by the informant, and its key points should be recorded in a daily
diary.
Statement 2 is correct. A zero FIR can be filed in any police station, regardless of whether the offence
was committed under the jurisdiction of that particular police station.
When a police station receives a complaint regarding an alleged offence that has been committed in the
jurisdiction of another police station, it registers an FIR, and then transfers it to the concerned police
station for further investigation. This is called a Zero FIR.
No regular FIR number is given. After receiving the Zero FIR, the concerned police station registers a
fresh FIR and starts the investigation.
Statement 3 is incorrect. All an informant has to do to file an FIR is to visit the local police station and
furnish information, orally or in writing, about the commission of an offence. Further, anyone can file an
FIR — the person approaching the police does not necessarily have to be the victim of or an eyewitness
to an offence.
Knowledge Base:
What is the difference between a complaint and an FIR?
The CrPC defines a “complaint” as “any allegation made orally or in writing to a Magistrate, with a view to
his taking action under this Code, that some person, whether known or unknown, has committed an
offence, but does not include a police report.”
However, an FIR is the document that has been prepared by the police after verifying the facts of the
complaint. The FIR may contain details of the crime and the alleged criminal.
Source: https://indianexpress.com/article/explained/everyday-explainers/fir-cognizable-offence-
ipc-explained-
7780266/#:~:text=In%20essence%20then%2C%20there%20are,should%20be%20recorded%20in%20a
https://ncert.nic.in/textbook.php?hess3=6-10

Q.57)
Ans) a
Exp) Option a is the correct answer
Article 28 of the Indian Constitution talks about Freedom as to attendance at religious instruction or
religious worship in certain educational institutions.
Option 1 is correct: Under Article 28 of the Indian Constitution, no religious instruction shall be
provided in any educational institution wholly maintained out of State funds. Hence religious
instructions are completely prohibited in educational institutions wholly maintained out of state funds.
Option 2 is incorrect: Religious instructions are permitted in educational institutions administered
by the State but established under any endowment or trust. Therefore there is no complete prohibition
of religious instruction in these institutions.
Option 3 and 4 are incorrect: As per Article 20 of the Indian Constitution, no person attending any
educational institution recognised by the State or receiving aid out of State funds shall be required to
attend any religious instruction or worship in that institution without his/her consent. There is no
complete prohibition of religious interactions as individuals are permitted to receive religious
instructions with their own consent.
Source: Laxmikanth - Chapter 7 (Fundamental Rights)

Q.58)
Ans) a
Exp) Option a is the correct answer.

Forum Learning Centre: Delhi - 2nd Floor, IAPL House, 19 Pusa Road, Karol Bagh, New Delhi - 110005 | Patna - 2nd floor, AG Palace, E Boring Canal
Road, Patna, Bihar 800001 | Hyderabad - 1st & 2nd Floor, SM Plaza, RTC X Rd, Indira Park Road, Jawahar Nagar, Hyderabad, Telangana 500020
9311740400, 9311740900 | https://academy.forumias.com | admissions@forumias.academy | helpdesk@forumias.academy
Page 27 of 47

PTS 2024 | B4 | L1 Test 1 - Solutions |


Fraternity means a sense of brotherhood. The Constitution promotes this feeling of fraternity among
citizens by the system of single citizenship.
Option a is correct: The Preamble of the Indian Constitution declares that fraternity has to assure two
things: the dignity of the individual and the unity and integrity of the nation. Therefore, it is true that
dignity of the individual and Unity and Integrity of the Nation are essential components of Fraternity. It
must be noted here that the word ‘integrity’ has been added to the preamble by the 42nd Constitutional
Amendment (1976).
Option b is incorrect: The Preamble of the Indian Constitution does not declare ensuring peace and
public order as the essential component of Fraternity.
Option c is incorrect: Elimination of Inequality is not the goal of Preamble however the Preamble aims
to secure all citizens of India a equality of status and opportunity to realise the objective of Equality (not
Fraternity)
Option d is incorrect: Equal treatment of all citizens without any discrimination based on caste, religion,
race, sec etc. is the essential component of Social Justice (not Fraternity).
Source: Laxmikanth - Chapter 4 (Preamble)

Q.59)
Ans) a
Exp) Option a is the correct answer
Article 32 confers the right to remedies for the enforcement of the fundamental rights of an aggrieved
citizen. Without this Article, Fundamental rights provided in the constitution would be meaningless. This
is the reason why Dr. B.R. Ambedkar called this article the ‘Soul of the Constitution’.
Statement 1 is incorrect: Article 32 empowers citizens of India to directly approach the Supreme Court
of India in case of violation of their fundamental rights only. Article 226 empowers citizens to directly
approach High courts in case of violation of their legal rights, fundamental right or any other rights.
Statement 2 is correct: The right to move the Supreme Court shall not be suspended except as otherwise
provided for by the Constitution. Thus, the Constitution provides that the President can suspend the
right to move any court for the enforcement of fundamental rights during a national emergency (Article
359).
Statement 3 is incorrect: The Parliament cannot curtail the powers conferred by Article 32 on the
Supreme Court. The Parliament can empower any other court to issue directions, orders and writs of all
kinds to enforce fundamental rights of citizens. However, this can be done without prejudice to the above
powers conferred on the Supreme Court.
Source: Laxmikanth - Chapter 7 (Fundamental Rights)

Q.60)
Ans) b
Exp) Option b is the correct answer.
A recent study said that turning the tropical forests into natural rubber plantations in Tripura is
negatively impacting non-human primate species and vegetation in the region. The report highlighted
that culturing natural rubber in the last century has brought significant economic benefits for growers.
But excessive rubber plantations are adversely affecting various wildlife and plant species.
Statement 1 is incorrect: Natural rubber is a polymer made up of a chemical molecule called isoprene. It
is a native of the Amazon basin which was introduced to countries in the tropical belts of Asia and
Africa in the late nineteenth century.
Statement 2 is incorrect: Rubber trees require moist and humid climates with heavy rainfall of more
than 200 cm. It grows well in equatorial climates and temperatures above 25 degrees Celsius.

Forum Learning Centre: Delhi - 2nd Floor, IAPL House, 19 Pusa Road, Karol Bagh, New Delhi - 110005 | Patna - 2nd floor, AG Palace, E Boring Canal
Road, Patna, Bihar 800001 | Hyderabad - 1st & 2nd Floor, SM Plaza, RTC X Rd, Indira Park Road, Jawahar Nagar, Hyderabad, Telangana 500020
9311740400, 9311740900 | https://academy.forumias.com | admissions@forumias.academy | helpdesk@forumias.academy
Page 28 of 47

PTS 2024 | B4 | L1 Test 1 - Solutions |

Rubber trees require well-drained, weathered soils.


Statement 3 is correct: Rubber is used for a variety of purposes from erasing pencil marks to
manufacturing tyres, tubes and a large number of industrial products. Natural rubber is preferred over
synthetic rubber due to its high tensile strength and vibration dampening properties, along with tear
resistance. This makes it important for the construction and automobile industries.
Statement 4 is correct: The Rubber Board is a statutory organization constituted under Section (4) of
the Rubber Act, 1947 and functions under the administrative control of Ministry of Commerce and
Industry. It is responsible for promotion and development of Rubber Industry in India.
Knowledge Base:
Rubber Board:
1) The Rubber Board is a statutory organization constituted under Section (4) of the Rubber Act, 1947
and functions under the administrative control of Ministry of Commerce and Industry.
2) The Board is headed by a chairman appointed by the Central Government and has 28 members
representing various interests of natural rubber industry.
3) The Board’s headquarters is located at Kottayam in Kerala.
4) The Board is responsible for the development of the rubber industry in the country by way of
assisting and encouraging research, development, extension, and training activities related to rubber.
5) It also maintains statistical data of rubber, takes steps to promote marketing of rubber and undertake
labour welfare activities.
6) The activities of the Board are exercised through Five Departments General Services, Extension &
Advisory Services, Research Services (rubber Research Institute of India), Training (Rubber Training
Institute) & Finance.
7) There are 5 independent divisions viz., Internal audit, Planning, Market Promotion, Publicity & Public
Relations, Vigilance.
Source: https://www.downtoearth.org.in/news/wildlife-biodiversity/rubber-plantations-in-tripura-
affecting-monkeys-vegetation-suggests-paper-86942
https://www.thehindu.com/news/national/explained-the-fall-in-natural-rubber-prices-in-
india/article65885937.ece

Q.61)
Ans) c
Exp) Option c is the correct answer
Article 23 of the Indian Constitution deals with Prohibition of Traffic in Human Beings and Forced
Labour.
Option 1 is incorrect: Although Article 23 prohibits traffic in human beings, begar (forced labour) and
other similar forms of forced labour, it does not define what constitutes human trafficking nor forced
labour. The Immoral Trafficking Act of 1956 lays down different aspects of trafficking and also prescribes
punishment for the same.
Option 2 and 3 are correct: It is true that Article 23 is available to both citizens and non-citizens. Also,
the article protects the individual not only against the State but also against private persons.
Option 4 is correct: Article 23 allows the State to impose compulsory service for public purposes, as for
example, military service or social service, for which state is not bound to pay. However, in imposing such
service, the State is not permitted to make any discrimination on grounds only of religion, race, caste or
class.
Source: Laxmikanth - Chapter 7 (Fundamental Rights)

Forum Learning Centre: Delhi - 2nd Floor, IAPL House, 19 Pusa Road, Karol Bagh, New Delhi - 110005 | Patna - 2nd floor, AG Palace, E Boring Canal
Road, Patna, Bihar 800001 | Hyderabad - 1st & 2nd Floor, SM Plaza, RTC X Rd, Indira Park Road, Jawahar Nagar, Hyderabad, Telangana 500020
9311740400, 9311740900 | https://academy.forumias.com | admissions@forumias.academy | helpdesk@forumias.academy
Page 29 of 47

PTS 2024 | B4 | L1 Test 1 - Solutions |

Q.62)
Ans) c
Exp) Option c is the correct answer.
The Constitution declares Delhi as the seat of the Supreme Court. The Supreme Court of India is the
highest judicial authority of India and it is located in New Delhi, India.
Option c is correct: The Constitution authorises the Chief Justice of India to appoint other places as a
seat of the Supreme Court with the consent of the President. This provision is only optional which
means that no court can give any direction either to the President or to the Chief Justice to appoint any
other place as a seat of the Supreme Court.
Source: Laxmikanth - Supreme Court

Q.63)
Ans) b
Exp) Option b is the correct answer
Statement 1 is correct: The Constitution does not define ‘religious denomination’. In a 1982 verdict, the
Supreme Court had defined religious denomination as “a religious sect or body having common faith
and organisation and designated by a distinctive name”. Hence the definition was provided by the
Supreme Court, not by the Constitution.
Statement 2 is correct: The Supreme Court held that a religious denomination must satisfy three
conditions:
a. It should be a collection of individuals who have a system of beliefs (doctrines) which they regard as
conducive to their spiritual well-being
b. It should have a common organisation
c. It should be designated by a distinctive name
Statement 3 is incorrect: The difference in essential religious practices is not necessary for a group to
be declared as a separate religious denomination. Essential religious practises are those practises which
are indispensable to the existence of religion i.e, without these practises the religion will not prevail.
Source: Laxmikanth - Chapter 7 (Fundamental Rights)

Q.64)
Ans) b
Exp) Option b is the correct answer
A Court of record is a trial court or appellate court in which a record of the proceedings is captured and
preserved, for the possibility of appeal. As a Court of Record, the Supreme Court has two powers:
1) The judgements of the Supreme Court are recorded for perpetual memory and testimony. These
records are admitted to be of evidentiary value and cannot be questioned when produced before
any court. They are recognised as legal precedents and legal references. Hence, Statement 1 is
correct.
2) As a court of record, the Supreme Court has power to punish for contempt of court, either with
simple imprisonment for a term up to six months or with fine up to ₹2,000 or with both. In 1991, the
Supreme Court ruled that it has power to punish for contempt not only of itself but also of high
courts, subordinate courts and tribunals functioning in the entire country. Hence, Statement 3 is
correct.
Statement 2 is incorrect: This power of the Supreme Court is related to its advisory jurisdiction. The
Constitution (Article 143) authorises the president to seek the opinion of the Supreme Court in the two
categories of matters:
a. On any question of law or fact of public importance which has arisen, or which is likely to arise.

Forum Learning Centre: Delhi - 2nd Floor, IAPL House, 19 Pusa Road, Karol Bagh, New Delhi - 110005 | Patna - 2nd floor, AG Palace, E Boring Canal
Road, Patna, Bihar 800001 | Hyderabad - 1st & 2nd Floor, SM Plaza, RTC X Rd, Indira Park Road, Jawahar Nagar, Hyderabad, Telangana 500020
9311740400, 9311740900 | https://academy.forumias.com | admissions@forumias.academy | helpdesk@forumias.academy
Page 30 of 47

PTS 2024 | B4 | L1 Test 1 - Solutions |

b. On any dispute arising out of any pre-constitution treaty, agreement, covenant, engagement, sanad
or other similar instruments.
Source: Laxmikanth - Supreme Court

Q.65)
Ans) b
Exp) Option b is the correct answer.
A new study on the Indian Star Tortoise (Geochelone elegans) has found that illegal trade and unscientific
translocations are causing major losses to the species' genetic diversity and habitat.
Statement 1 is incorrect: Indian star tortoises are found across the Indian subcontinent, more
specifically, in the Central and Southern parts of India, in West Pakistan and in Sri Lanka.
Statement 2 is correct: Indian star tortoises is typically found in dry, open habitats such as scrub
forests, grasslands, and rocky outcroppings.
Statement 3 is incorrect: They are categorized as ‘Vulnerable’ in IUCN Red list. Convention on
International Trade in Species (CITES): Appendix I
Statement 4 is correct: The species is facing twin challenges of a threat to its habitat at one level and
loss of its genetic diversity at the other. Their highly fragmented habitat of the species is greatly
influenced by an increased level of urbanization and agricultural practices. Due to subsequent
hybridization of these species over the years, Indian star tortoises have lost genetic diversity. Also,
according to the Wildlife Crime Control Bureau, 90% of the trade of Star Tortoise occurs as part of the
international pet market.
Source: https://www.thehindu.com/sci-tech/energy-and-environment/indian-star-tortoise-faces-
twin-challenges-of-habitat-loss-and-genetic-diversity-finds-study/article66397486.ece

Q.66)
Ans) a
Exp) Option a is the correct answer.
A doctrine is a principle, belief, or position, often held by authorities such as courts. A doctrine can be a
rule, a theory or a tenet of law. There are many judicial doctrines applied under the Constitution of India.
Pair 1 is incorrect: The Doctrine of Eclipse states that any law which is inconsistent with fundamental
rights does not completely become invalid. It is not totally dead but overshadowed by the fundamental
right. The inconsistency (conflict) can be removed by constitutional amendment. The amendment to the
relevant fundamental right will remove the eclipse and the entire law becomes valid. Doctrine of eclipse
is contended in Article 13(1) of the Indian Constitution.
The Doctrine of Ancillary Powers implies that the power to legislate on a topic of legislation carries with
it the power to legislate on an ancillary matter which can be said to be reasonably included in the power
given.
Pair 2 is correct: The phrase “Pith and Substance” means true nature and character. This Doctrine of
Pith and Substance determines whether a specific law relating to a particular subject falls within the
legitimate power of a legislature. In such cases, the court looks at the substance of the matter. Thus, for
example, if the substance falls within Union List, then the incidental encroachment by the law on the
State List does not make it invalid.
Pair 3 is incorrect: Doctrine of Colourable Legislation means when a legislature does not have the
power to make laws on a particular subject directly, it cannot make laws on it indirectly.
Pair 4 is incorrect: Doctrine of Severability is also known as the Doctrine of Separability. It states that if
any part of law is inconsistent with the fundamental rights, only that part shall be treated by the
courts as void, not the whole law.

Forum Learning Centre: Delhi - 2nd Floor, IAPL House, 19 Pusa Road, Karol Bagh, New Delhi - 110005 | Patna - 2nd floor, AG Palace, E Boring Canal
Road, Patna, Bihar 800001 | Hyderabad - 1st & 2nd Floor, SM Plaza, RTC X Rd, Indira Park Road, Jawahar Nagar, Hyderabad, Telangana 500020
9311740400, 9311740900 | https://academy.forumias.com | admissions@forumias.academy | helpdesk@forumias.academy
Page 31 of 47

PTS 2024 | B4 | L1 Test 1 - Solutions |


Source: https://www.legalserviceindia.com/legal/article-4535-doctrine-of-eclipse.html
https://legalserviceindia.com/legal/article-4014-doctrine-of-severability-a-scalpel-rather-than-a-
bulldozer.html
https://www.legalserviceindia.com/legal/article-6798-doctrine-of-pith-and-substance.html

Q.67)
Ans) a
Exp) Option a is the correct answer.
Articles 214 to 231 in Part VI of the Constitution deal with the organisation, independence, jurisdiction,
powers, procedures and so on of the high courts.
Statement 1 is correct: A judge of a high court can be removed from his office by an order of the
President. The President can issue the removal order only after an address by the Parliament has been
presented to him in the same session for such removal.
Statement 2 is incorrect: The grounds of removal are two—proved misbehaviour or incapacity.
Adjudged an insolvent is not a criterion for removal of judge.
Statement 3 is incorrect: The procedure for removal of judges is elaborated in the Judges Inquiry Act,
1968. Under the Act, an impeachment motion may originate in either House of Parliament. To initiate
proceedings: (1) at least 100 members of Lok Sabha may give a signed notice to the Speaker, or (2) at least
50 members of Rajya Sabha may give a signed notice to the Chairman. The Speaker/Chairman may
decide to either admit the motion or refuse to admit it.
Source: Ch-34, High Court, Laxmikanth
https://prsindia.org/theprsblog/explainer-removal-of-judges-from-
office#:~:text=The%20Constitution%20provides%20that%20a,the%20Judges%20Inquiry%20Act%2C%20
1968.

Q.68)
Ans) b
Exp) Option b is the correct answer.
The 44th Amendment Act of 1978 abolished the right to property as a Fundamental Right by repealing
Article 19(1)(f) and Article 31 from Part III. Instead, the Act inserted a new Article 300A in Part XII under
the heading ‘Right to Property’.
Statement 1 and 4 are correct: Though the Fundamental Right to Property under Part III has been
abolished, the Part III still carries two provisions which provide for the guaranteed right to
compensation in case of acquisition or requisition of the private property by the state. These two cases
where compensation has to be paid are:
1) When the State acquires the property of a minority educational institution (Article 30); and
2) When the State acquires the land held by a person under his personal cultivation and the land is
within the statutory ceiling limits (Article 31 A).
Statement 2 and 3 are incorrect: There is no such provision given in the Constitution for
compensation for the land acquired from a person belongs to the Scheduled Tribe (ST) or Scheduled
Caste (SC).
Source: Ch-7, Fundamental Rights, Laxmikanth

Q.69)
Ans) c
Exp) Option c is the correct answer.

Forum Learning Centre: Delhi - 2nd Floor, IAPL House, 19 Pusa Road, Karol Bagh, New Delhi - 110005 | Patna - 2nd floor, AG Palace, E Boring Canal
Road, Patna, Bihar 800001 | Hyderabad - 1st & 2nd Floor, SM Plaza, RTC X Rd, Indira Park Road, Jawahar Nagar, Hyderabad, Telangana 500020
9311740400, 9311740900 | https://academy.forumias.com | admissions@forumias.academy | helpdesk@forumias.academy
Page 32 of 47

PTS 2024 | B4 | L1 Test 1 - Solutions |

Option 4: India conducted its first nuclear tests on May 18, 1974, in Pokhran, Rajasthan, as part of the
'Smiling Buddha' operation.
Option 3: The Sikkim finally opted to become full fledged State of the Indian Union with effect from 26
April 1975 vide the Constitution 36th Amendment Act 1975 with special provision laid for the State under
article 371(F) of the Constitution of India.
Option 2: The Tenth Schedule was inserted in the Constitution in the year 1985 through 52nd
Constitutional Amendment Act. It introduced anti-defection law in India. This amendment was to curb
political defections motivated by lure of office or other similar considerations which endanger the
foundations of our democracy.
Option 1: The Sixty-first Amendment of the Constitution of India, officially known as The Constitution
(Sixty-first Amendment) Act, 1989, lowered the voting age of elections to the Lok Sabha and to the
Legislative Assemblies of States from 21 years to 18 years.
Hence, correct order is 4 – 3- 2 - 1.
Source: Chapter 5, Union and its Territory, Laxmikant
https://timesofindia.indiatimes.com/india/how-india-pakistan-war-of-1971-started-how-we-won-
significance-of-vijay-
diwas/articleshow/96271054.cms#:~:text=Vijay%20Diwas%2C%20which%20is%20celebrated,soldiers%2
0who%20defended%20the%20nation.

Q.70)
Ans) b
Exp) Option b is the correct answer.
Recently, Hindenburg Research has accused the Adani Group of fraud of short selling.
Short selling is a trading strategy where an investor borrows shares of a stock they believe will decline
in value, sells the stock, and then buys the stock back at a lower price to make a profit. In other words,
the investor is "betting against" the stock, as they are making money when the price of the stock goes
down. While fundamentally it is based on the “buy low, sell high” approach, the sequence of transactions
is reversed in short selling — to sell high first and buy low later. Also, in short selling, the trader usually
does not own the securities he sells, but merely borrows them.
• To engage in short selling, an investor must first find a lender that is willing to lend the shares. The
lender will usually require collateral from the investor to cover any potential losses if the price of the
stock goes up instead of down.
• If the price of the stock does decline as the investor expects, they can buy the shares back at the
lower price, return the borrowed shares to the lender, and pocket the difference as profit.
Source: https://indianexpress.com/article/explained/everyday-explainers/hindenburg-research-
accused-the-adani-fraud-short-seller-8406285/

Q.71)
Ans) b
Exp) Option b is the correct answer.
Economic justice denotes the non-discrimination between people on the basis of economic factors. It
involves the elimination of glaring inequalities in wealth, income and property.
Option 1 is correct: Preamble states Justice, Liberty, Equality and Fraternity as objectives of the Indian
Constitution. The term ‘justice’ in the Preamble embraces three distinct forms– social, economic and
political justice. Thus Preamble reflects the principles of Economic Justice.
Option 2 is correct: Directive Principles of State Policy (DPSPs) are enumerated in Part IV (Article 36-51)
of the Indian Constitution. The DPSPs are meant for promoting the ideal of social and economic justice,

Forum Learning Centre: Delhi - 2nd Floor, IAPL House, 19 Pusa Road, Karol Bagh, New Delhi - 110005 | Patna - 2nd floor, AG Palace, E Boring Canal
Road, Patna, Bihar 800001 | Hyderabad - 1st & 2nd Floor, SM Plaza, RTC X Rd, Indira Park Road, Jawahar Nagar, Hyderabad, Telangana 500020
9311740400, 9311740900 | https://academy.forumias.com | admissions@forumias.academy | helpdesk@forumias.academy
Page 33 of 47

PTS 2024 | B4 | L1 Test 1 - Solutions |

for instance Article 39(d) stipulates that the state shall endeavour to secure equal pay for equal work for
both men and women.
Option 3 is incorrect: The Part IV-A of the Constitution (Article 51-A) specifies eleven Fundamental
Duties of Indian Citizens such as to respect the Constitution, national flag and national anthem etc. The
fundamental duties do not provide for economic justice as it serves mainly as a reminder to citizens that
while enjoying their rights, they have also to be conscious of duties they owe to their country, their
society and to their fellow citizens.
Source: Laxmikanth - Chapter 3 (Salient Features of the Constitution)

Q.72)
Ans) a
Exp) Option a is the correct answer
There are certain reasonable classification where restrictions upon Fundamental Rights may be
imposed upon some of the citizens, such as members of the Armed Forces or the members of the Forces
charged with the maintenance of public order etc.
Statement 1 is incorrect: Article 33 empowers the Parliament both to restrict or abrogate the
fundamental rights of the members of armed forces, para-military forces, police forces, intelligence
agencies and analogous forces. The objective of this provision is to ensure the proper discharge of their
duties and the maintenance of discipline among them.
Statement 2 is correct: The forces on which the restriction of fundamental rights can be applied are
armed forces, para-military forces, police forces, and similar agencies. It can also be the members of
the forces charged with the maintenance of public order or persons employed in any bureau or other
organisation established by the State for purposes of intelligence or counter intelligence or persons
employed in, or in connection with, the telecommunication systems set up for the purposes of any Force,
bureau or organisation
Statement 3 is incorrect: The power to make laws under Article 33 is conferred only on Parliament and
not on state legislatures. Any such law made by Parliament cannot be challenged in any court on the
ground of contravention of any of the Fundamental Rights.
Source: Indian Polity by Laxmikanth, 6th Edition; Chapter-7
https://www.legalserviceindia.com/legal/article-1913-do-members-of-armed-forces-in-india-enjoy-
their-fundamental-rights.html

Q.73)
Ans) b
Exp) Option b is the correct answer.
The High Courts in India are the highest courts of appellate jurisdiction in each state and union territory.
Statement 1 is correct: Unlike in the case of the Supreme Court, the Constitution makes no provision
for appointment of a distinguished jurist as a judge of a High Court.
Statement 2 is correct and Statement 3 is incorrect:
Qualification to become the Judge of the High Court are-
1) He should be a citizen of India.
2) (a) He should have held a judicial office in the territory of India for ten years; or (b) He should have
been an advocate of a High court (or High courts in succession) for ten years.
Source: Laxmikanth (Ch 34)

Forum Learning Centre: Delhi - 2nd Floor, IAPL House, 19 Pusa Road, Karol Bagh, New Delhi - 110005 | Patna - 2nd floor, AG Palace, E Boring Canal
Road, Patna, Bihar 800001 | Hyderabad - 1st & 2nd Floor, SM Plaza, RTC X Rd, Indira Park Road, Jawahar Nagar, Hyderabad, Telangana 500020
9311740400, 9311740900 | https://academy.forumias.com | admissions@forumias.academy | helpdesk@forumias.academy
Page 34 of 47

PTS 2024 | B4 | L1 Test 1 - Solutions |

Q.74)
Ans) b
Exp) Option b is the correct answer.
The right to freedom guarantees freedom for citizens to live a life of dignity among other things. These
are given in Articles 19, 20, 21A and 22 of the Indian Constitution.
Article 21 of the Constitution provides that no person shall be deprived of his life and personal liberty
except according to procedure established by law, which is inbuilt guarantee against torture or assault
by the state or its functionaries including ‘Right against custodial harassment’.
Source: Ch-7, Fundamental rights, Laxmikanth

Q.75)
Ans) a
Exp) Option a is the correct answer.
According to a new study published in Nature Geosciences, the nighttime production of nitrate
radicals could make it difficult to improve India's air quality. A new study has found that parts of India
and China are hotspots for the night-time production of nitrate radicals that can increase the amount of
deadly ozone and PM2.5 particulate matter in the atmosphere.
Both Statement 1 and Statement 2 are correct and Statement 2 is the correct explanation of Statement
1:
Nitrate radical is an oxide of nitrogen that consists of three oxygen atoms bound to a nitrogen atom.
Nitrogen oxides are reactive gases that regulate the formation of air pollutants, including ozone and
PM2.5 particles. Nitrate radicals will oxidize gas pollutants such as volatile organic compounds (VOCs),
which will then generate ozone and secondary organic aerosol. Ozone is an air pollutant that affects
human health and crop yield. Secondary organic aerosol is an important component of PM2.5
Source: https://indianexpress.com/article/technology/nitrate-radical-production-pollution-india-
china-8406729/

Q.76)
Ans) b
Exp) Option b is the correct answer.
There are many forms of government in the world. One major form of classification is on the basis of the
extent of separation of powers between the Executive and the Legislature, and the features emanating
from such a difference. On this basis there are 2 types of governments - Parliamentary (like in India, UK,
etc) and Presidential (like in USA, Brazil, Russia, etc) forms of government. Following are some
distinguishing features of Parliamentary form of government.
Statement 1 is incorrect: It is in the Presidential form of government, not the Parliamentary form of
government, that there is a strict Separation of Powers.
Strict Separation of Powers means that there is no overlap in the membership and functions of the 3
organs of government - the Executive, the Legislative, and the Judiciary. In the Parliamentary form of
government, as seen in India, and the UK, the Executive (the Prime Minister and his Council of
Ministers (CoM)), compulsorily have to be members of the Legislature. So, there is an overlap of
membership.
Statement 2 is correct: In the Parliamentary form of government, one of the most basic and important
features is that of Collective Responsibility. This means that the Executive (the PM and his CoM), are
members of Parliament, and answerable to the Legislature for their acts of omission and commission.
So, if they lose the confidence (their majority) of the Lok Sabha, they are liable to be removed. And it

Forum Learning Centre: Delhi - 2nd Floor, IAPL House, 19 Pusa Road, Karol Bagh, New Delhi - 110005 | Patna - 2nd floor, AG Palace, E Boring Canal
Road, Patna, Bihar 800001 | Hyderabad - 1st & 2nd Floor, SM Plaza, RTC X Rd, Indira Park Road, Jawahar Nagar, Hyderabad, Telangana 500020
9311740400, 9311740900 | https://academy.forumias.com | admissions@forumias.academy | helpdesk@forumias.academy
Page 35 of 47

PTS 2024 | B4 | L1 Test 1 - Solutions |

also means that if the PM (the head of the Executive) dies or is removed, the entire remaining Executive
stands removed.
Statement 3 is correct: In a Parliamentary form of government, the Executive is responsible to the
Legislative. So, a government can stay in power only so long as it enjoys a majority in the Lower House
of the Parliament. In the case, that the government of the day ceases to hold this majority, showcased by
a vote of no confidence, or the Prime Minister himself feels so, and wants to call for fresh elections, the
President has the power to dissolve the Lower House of the Parliament (Lok Sabha) and order fresh
elections.
This is not the case in the Presidential form of government. There is no requirement for the political
party to which the President belongs to have a majority in Congress (Lower House) as well. So, there is
no question of no confidence or dissolution. So, the Lower House in such polities is not dissolved by the
President.
Statement 4 is incorrect: Parliamentary form of government is characterised by a Dual Executive. This
means that there is a nominal Executive head - the President, in whose name most of the executive
decisions and actions are performed; and there is the real Executive Head - the Prime Minister, in
whose hands, the real Executive power lies. While the President is the Head of State, and possesses
some powers in name and some in reality, the Prime Minister is the Head of the Government, and
possesses many powers for all practical purposes, even if not in name.
Source: Indian Polity by Laxmikanth, 5th edition, Ch-12, Pg-12.3, 12.4

Q.77)
Ans) b
Exp) Option b is the correct answer.
Pair 1 is incorrect: Habeas Corpus-
1) It is a Latin term which literally means ‘to have the body of’. It is an order issued by the court to a
person who has detained another person, to produce the body of the latter before it. The court then
examines the cause and legality of detention. It would set the detained person free, if the detention is
found to be illegal. Thus, this writ is a bulwark of individual liberty against arbitrary detention.
2) The writ of habeas corpus can be issued against both public authorities as well as private individuals.
3) The writ, on the other hand, is not issued where the (a) detention is lawful, (b) the proceeding is for
contempt of a legislature or a court, (c) detention is by a competent court, and (d) detention is outside
the jurisdiction of the court.
Pair 2 is incorrect: Mandamus-
1) It literally means ‘we command’. It is a command issued by the court to a public official asking him to
perform his official duties that he has failed or refused to perform.
2) It can also be issued against any public body, a corporation, an inferior court, a tribunal or government
for the same purpose.
3) The writ of mandamus cannot be issued (a) against a private individual or body; (b) to enforce
departmental instruction that does not possess statutory force; (c) when the duty is discretionary and
not mandatory; (d) to enforce a contractual obligation; (e) against the president of India or the state
governors; and (f) against the chief justice of a high court acting in judicial capacity.
Pair 3 is correct: Prohibition-
1) Literally, it means ‘to forbid’. It is issued by a higher court to a lower court or tribunal to prevent the
latter from exceeding its jurisdiction or usurping a jurisdiction that it does not possess. Thus, unlike
mandamus that directs activity, the prohibition directs inactivity.
2) The writ of prohibition can be issued only against judicial and quasi judicial authorities. It is not
available against administrative authorities, legislative bodies, and private individuals or bodies.

Forum Learning Centre: Delhi - 2nd Floor, IAPL House, 19 Pusa Road, Karol Bagh, New Delhi - 110005 | Patna - 2nd floor, AG Palace, E Boring Canal
Road, Patna, Bihar 800001 | Hyderabad - 1st & 2nd Floor, SM Plaza, RTC X Rd, Indira Park Road, Jawahar Nagar, Hyderabad, Telangana 500020
9311740400, 9311740900 | https://academy.forumias.com | admissions@forumias.academy | helpdesk@forumias.academy
Page 36 of 47

PTS 2024 | B4 | L1 Test 1 - Solutions |

Pair 4 is correct: Quo-Warranto-


1) In the literal sense, it means ‘by what authority or warrant’. It is issued by the court to enquire into the
legality of claim of a person to a public office. Hence, it prevents illegal usurpation of public office by a
person.
2) The writ can be issued only in case of a substantive public office of a permanent character created by
a statute or by the Constitution.
3) It cannot be issued in cases of ministerial office or private office. Unlike the other four writs, this can
be sought by any interested person and not necessarily by the aggrieved person.
Source: chapter 7, m laxmikant 5th edition

Q.78)
Ans) c
Exp) Option c is the correct answer.
The Supreme Court has numerous powers including deciding the disputes regarding the election of the
President and the Vice-President, conducting enquiry into the conduct of the chairman and members of
the Union Public Service Commission on a reference made by the President, etc.
Statement 1 is correct: The law of the Supreme Court is binding on all courts in India. Its decree or
order is enforceable throughout the country. Thus, all authorities (civil and judicial) in the country
should act in aid of the Supreme Court.
Statement 2 is correct: The Supreme Court has power of judicial superintendence and control over all
the courts and tribunals functioning in the entire territory of the country.
Statement 3 is correct: The Supreme Court can transfer a case or appeal pending before one High
Court to another high court. It is also authorised to withdraw the cases pending before the high courts
and dispose them by itself.
Source: Indian Polity, M. Laxmikanth, Chapter-26, Supreme Court

Q.79)
Ans) b
Exp) Option b is the correct answer.
The members of the Constituent Assembly were elected by the provincial assemblies by a single,
transferable-vote system of proportional representation. The total membership of the Constituent
Assembly was 389 of which 292 were representatives of the provinces, 93 represented the princely
states and four were from the chief commissioner provinces of Delhi, Ajmer-Merwara, Coorg and British
Baluchistan.
Statement 1 is incorrect. The Congress Party held a large majority in the Constituent Assembly (69
percent of the seats), and the Muslim League held nearly all the seats reserved in the Assembly for
Muslims. There were also members of smaller parties, such as the Scheduled Caste Federation, the
Communist Party of India and the Unionist Party. Communist Party of India got 1 seat in the
Constituent Assembly.
Statement 2 is correct. Muslim League with 73 seats was the second largest party after Indian National
Congress (208 seats) in the Constituent Assembly of India.

Forum Learning Centre: Delhi - 2nd Floor, IAPL House, 19 Pusa Road, Karol Bagh, New Delhi - 110005 | Patna - 2nd floor, AG Palace, E Boring Canal
Road, Patna, Bihar 800001 | Hyderabad - 1st & 2nd Floor, SM Plaza, RTC X Rd, Indira Park Road, Jawahar Nagar, Hyderabad, Telangana 500020
9311740400, 9311740900 | https://academy.forumias.com | admissions@forumias.academy | helpdesk@forumias.academy
Page 37 of 47

PTS 2024 | B4 | L1 Test 1 - Solutions |

Source: Laxmikanth. Chapter 2

Q.80)
Ans) a
Exp) Option a is the correct answer.
Rules of Procedure and Conduct of Business in the Council of States (Rajya Sabha) have been formulated
under Article 118 of the Constitution wherein each House of Parliament is required to make rules for
regulating its procedure and conduct of business.
Statement a is correct: Under the Rule 267, the Rule gives special power to a Rajya Sabha member to
suspend the pre-decided agenda of the House, with the approval of the Chairman. It says,” Any member,
may, with the consent of the Chairman, move that any rule may be suspended in its application to a
motion related to the business listed before the Council of that day and if the motion is carried, the rule
in question shall be suspended for the time being.”
Statement b is incorrect: Rule 51A of the Rules of Procedure and Conduct of Business in the Rajya Sabha
states regarding reduction in the limit of number of questions for oral answers to 15 from existing 20.
Whereas, Rule 267 says about the suspension of the pre-decided agenda in the council of the states.
Statement c is incorrect: Rule 256 of the rules of Procedure and Conduct of Business provides
suspension of member (1) The Chairman may, if he deems it necessary, name a member who disregards
the authority of the Chair or abuses the rules of the Council by persistently and willfully obstructing the
business thereof. (2) If a member is so named by the Chairman, he shall forthwith put the question on a
motion being made, no amendment, adjournment or debate being allowed, that the member (naming him)
be suspended from the service of the Council for a period not exceeding the remainder of the Session:
Provided that the Council may, at any time, on a motion being made, resolve that such suspension be
terminated.
Statement d is incorrect: Rule 257 of the rules of Procedure and Conduct of Business in the Rajya Sabha
provides power of Chairman to adjourn Council or suspend sitting in the case of grave disorder arising in
the Council. Whereas, Whereas, Rule 267 says about the suspension of the pre-decided agenda in the
council of the states.
Source: https://www.indiatimes.com/explainers/news/what-is-rule-267-of-rajya-sabha-and-why-
was-it-in-the-discussion-recently-588148.html
2510RS (Pre).p65 (rajyasabha.nic.in)

Q.81)
Ans) c
Exp) Option c is the correct answer

Forum Learning Centre: Delhi - 2nd Floor, IAPL House, 19 Pusa Road, Karol Bagh, New Delhi - 110005 | Patna - 2nd floor, AG Palace, E Boring Canal
Road, Patna, Bihar 800001 | Hyderabad - 1st & 2nd Floor, SM Plaza, RTC X Rd, Indira Park Road, Jawahar Nagar, Hyderabad, Telangana 500020
9311740400, 9311740900 | https://academy.forumias.com | admissions@forumias.academy | helpdesk@forumias.academy
Page 38 of 47

PTS 2024 | B4 | L1 Test 1 - Solutions |

Statement I is correct: It is true that the Preamble mentions India as a Secular state. The term ‘secular’
was added to the Preamble of the Indian Constitution by the 42nd Constitutional Amendment Act of 1976.
The Preamble secures to all citizens of India liberty of belief, faith and worship.
Statement II is incorrect: It is true that secularism means complete separation of religion from the
affairs of state as adopted by Western countries. However, the Indian model of Secularism does not
mean complete separation of religion and state as State intervenes in religious matter to protect the
dignity if individuals eg. abolition of untouchability. Hence Statement II is not the correct explanation
of Statement I.
Source: https://ncert.nic.in/ncerts/l/keps108.pdf

Q.82)
Ans) c
Exp) Option c is the correct answer
On August 20, 1917, the British Government declared that its objective was the gradual introduction of
responsible Government in India. Eventually the Government of India Act of 1919 was enacted to enable
responsible government in India. This Act is also known as Montagu- Chelmsford Reforms (Montagu was
the Secretary of State for India and Lord Chelmsford was the Viceroy of India).
Statement 1 is correct: The Government of India Act, 1919, for the first time introduced direct elections
in the country. Also it granted franchise to a limited number of people on the basis of property, tax or
education. The majority of members of both the Houses of Indian Legislative Council were chosen by
direct election.
Statement 2 is correct: The Government of India Act, 1919 introduced, for the first time, bicameral
legislature at Centre. Thus, the Indian legislative council was replaced by a bicameral legislature
consisting of an Upper House (Council of State) and a Lower House (Legislative Assembly).
Statement 3 is correct: It is true that the Government of India Act, 1919 separated, for the first time,
provincial budgets from the Central budget and authorised the provincial legislatures to enact their
own budgets.
Source: Laxmikanth - Chapter 1 (Historical Background)

Q.83)
Ans) a
Exp) Option a is the correct answer.
Articles 214 to 231 in Part VI of the Constitution deal with the organisation, independence, jurisdiction,
powers, and procedures and so on of the high courts.
Statement 1 is correct: The territorial jurisdiction of a high court is co-terminus with the territory of a
state. Similarly, the territorial jurisdiction of a common high court is co-terminus with the territories of
the concerned states and union territory.
Statement 2 is incorrect: The Constitution of India provides for a high court for each state, but the 7th
Amendment Act of 1956 authorised the Parliament to establish a common high court for two or more
states or for two or more states and a union territory. At present, there are 25 high courts in the country.
Out of them, only three high courts have jurisdiction over more than one state.
Statement 3 is incorrect: The Parliament (and not the Supreme Court) can extend the jurisdiction of a
high court to any union territory or exclude the jurisdiction of a high court from any union territory.
Among the nine union territories, Delhi alone has a separate high court (since 1966). The union territories
of Jammu and Kashmir and Ladakh have a common high court. The other union territories fall under the
jurisdiction of different state high courts.
Source: Indian Polity, M. Laxmikanth, Chapter-34, High Court

Forum Learning Centre: Delhi - 2nd Floor, IAPL House, 19 Pusa Road, Karol Bagh, New Delhi - 110005 | Patna - 2nd floor, AG Palace, E Boring Canal
Road, Patna, Bihar 800001 | Hyderabad - 1st & 2nd Floor, SM Plaza, RTC X Rd, Indira Park Road, Jawahar Nagar, Hyderabad, Telangana 500020
9311740400, 9311740900 | https://academy.forumias.com | admissions@forumias.academy | helpdesk@forumias.academy
Page 39 of 47

PTS 2024 | B4 | L1 Test 1 - Solutions |

Q.84)
Ans) a
Exp) Option a is the correct answer.
Statement 1 is incorrect: Article 29 provides for the ‘Protection of Interest of Minorities’
1) Article 29 provides that any section of the citizens residing in any part of India having a distinct
language, script or culture of its own, shall have the right to conserve the same.
2) Further, no citizen shall be denied admission into any educational institution maintained by the
State or receiving aid out of State funds on grounds only of religion, race, caste, or language
Article 29 grants protection to both religious minorities as well as linguistic minorities. However, the
Supreme Court held that the scope of this article is not necessarily restricted to minorities only, as it
is commonly assumed to be. This is because of the use of words ‘section of citizens’ in the Article that
include minorities as well as majority.
Statement 2 is correct: The Supreme Court also held that the right to conserve the Language under
Article 29 includes the right to agitate for the protection of the language. Hence, the political speeches
or promises made for the conservation of the language of a section of the citizens does not amount to
corrupt practice under the Representation of the People Act, 1951.
Statement 3 is incorrect: Article 30 provides for the Right of Minorities to Establish and Administer
Educational Institutions. It grants the following rights to minorities, whether religious or linguistic:
1) All minorities shall have the right to establish and administer educational institutions of their
choice.
2) The compensation amount fixed by the State for the compulsory acquisition of any property of a
minority educational institution shall not restrict or abrogate the right guaranteed to them.
3) In granting aid, the State shall not discriminate against any educational institution managed by a
minority
Thus, the protection under Article 30 is confined only to minorities (religious or linguistic) and does
not extend to any section of citizens (as under Article 29).
Knowledge Base: The Constitution of India does not define the word "minorities".
Source: Chapter7: Fundamental Rights of Indian Polity by M.Laxmikant

Q.85)
Ans) a
Exp) Option a is the correct answer.
The World Trade Organisation (WTO) Agreement on Fisheries Subsidies adopted at the 12th Ministerial
Conference (MC12) held in Geneva, Switzerland in 2022 and the agreement aims to protect ocean
sustainability by prohibiting harmful fisheries subsidies, which are a key factor in the widespread
depletion of the world’s fish stocks.
Statement 1 is correct: At least two-thirds of members have to deposit their instruments of acceptance
with the WTO, for the New Agreement on Fisheries Subsidies to take effect.
Statement 2 is incorrect: As part of special and differential treatment (S&DT), developing countries like
India have been given a two-year transition period for eliminating subsidies given to illegal, unreported,
or unregulated (IUU) fishing and fishing of already over-exploited stocks. However, India demands a 25-
year transition period to protect the livelihood of people depending on the fishery sector in India.
Statement 3 is incorrect: Recently, Switzerland became the first WTO member to formally submit its
instrument of acceptance for the New Agreement of Fisheries subsidies with the WTO. India did not
provide her acceptance for this agreement so far.
Source: https://www.wto.org/english/res_e/booksp_e/impfishag_part_2_e.pdf
https://indianexpress.com/article/opinion/columns/the-wto-agreement-on-fisheries-is-flawed-but-
significant-7985390/

Forum Learning Centre: Delhi - 2nd Floor, IAPL House, 19 Pusa Road, Karol Bagh, New Delhi - 110005 | Patna - 2nd floor, AG Palace, E Boring Canal
Road, Patna, Bihar 800001 | Hyderabad - 1st & 2nd Floor, SM Plaza, RTC X Rd, Indira Park Road, Jawahar Nagar, Hyderabad, Telangana 500020
9311740400, 9311740900 | https://academy.forumias.com | admissions@forumias.academy | helpdesk@forumias.academy
Page 40 of 47

PTS 2024 | B4 | L1 Test 1 - Solutions |

Q.86)
Ans) a
Exp) Option a is the correct answer.
State Administrative Tribunals (SATs) are specialized quasi-judicial bodies established at the state level in
India. These tribunals are designed to provide an alternative forum for resolving disputes and grievances
related to the recruitment, conditions of service, and other administrative matters of state government
employees.
Statement 1 is incorrect: The Administrative Tribunals Act of 1985 empowers the Central government to
establish the State Administrative Tribunals (SATs) on specific request of the concerned state
governments.
Statement 2 is correct: SATs exercise original jurisdiction in relation to recruitment and all service
matters of state government employees. This means that once a dispute or grievance is filed before the
SAT, it cannot be adjudicated by any other court or tribunal, except for appeals to higher courts.
Statement 3 is incorrect: The chairman and members of the SATs are appointed by the president after
consultation with the governor of the state concerned. SATs are typically composed of administrative
members and judicial members. The administrative members are usually senior bureaucrats or retired
civil servants, while the judicial members are retired judges or legal professionals with relevant expertise.
Source: Laxmikanth Ch. 63 Tribunals

Q.87)
Ans) b
Exp) Option b is the correct answer.
Statement 1 is correct. The Bar Council of India was established by Parliament under the Advocates Act,
1961. It regulates and represents the Indian bar. It safeguards the rights, privileges and interests of
advocates.
Statement 2 is incorrect. It exercises disciplinary jurisdiction over advocates/lawyers. It performs the
regulatory function by prescribing standards of professional conduct and etiquette and by exercising
disciplinary jurisdiction over the bar. It lays down procedure to be followed by its disciplinary committee
and the disciplinary committees of each State Bar Council.
Statement 3 is correct. It grants recognition to universities whose degree in law shall be a qualification
for enrolment as an advocate. The Bar Council of India visits and inspects Universities, or directs the
State Bar Councils to visit and inspect Universities for this purpose.
It promotes legal education and lays down standards of legal education. This is done in consultation with
the Universities in India imparting legal education and the State Bar Councils
Source: http://www.barcouncilofindia.org/about/about-the-bar-council-of-india/

Q.88)
Ans) a
Exp) Option a is the correct answer.
According to Article 1, the territory of India can be classified into three categories: 1. Territories of the
states 2. Union territories 3. Territories that may be acquired by the Government of India at any time.
Statement 1 is correct: Notably, the ‘Territory of India’ is a wider expression than the ‘Union of India’.
Statement 2 is correct: The ‘Union of India’ includes only states while the ‘Territory of India’ includes not
only the states but also union territories and territories that may be acquired by the Government of India
at any future time.
Since, both statements are correct and statement 2 is correct explanation of statement 1. Hence,
option a is correct answer.
Source: Chapter 5, Union and its Territory, Laxmikant

Forum Learning Centre: Delhi - 2nd Floor, IAPL House, 19 Pusa Road, Karol Bagh, New Delhi - 110005 | Patna - 2nd floor, AG Palace, E Boring Canal
Road, Patna, Bihar 800001 | Hyderabad - 1st & 2nd Floor, SM Plaza, RTC X Rd, Indira Park Road, Jawahar Nagar, Hyderabad, Telangana 500020
9311740400, 9311740900 | https://academy.forumias.com | admissions@forumias.academy | helpdesk@forumias.academy
Page 41 of 47

PTS 2024 | B4 | L1 Test 1 - Solutions |

Q.89)
Ans) d
Exp) Option d is the correct answer.
A quasi-judicial body is a body which has powers and procedures resembling those of a court of law or
judge such as an arbitrator or tribunal board. It is obliged to objectively determine facts and draw
conclusions from them so as to provide the basis of an official action.
Option 1 is correct. The National Consumer Disputes Redressal Commission (NCDRC) of India is a
quasi-judicial commission in India which was set up in 1988 under the Consumer Protection Act, 1986. Its
head office is in New Delhi. The commission is headed by a sitting or retired judge of the Supreme Court
of India.
Option 2 is correct. National Human right commission: National Human Rights Commission is a quasi-
judicial body which looks into cases of specifically Human Rights violation. It was established under the
Human right act 1993. They can investigate human right abuse and can recommend the steps to be taken.
Option 3 is correct. The Central Information Commission is a statutory body and quasi-judicial body
set up under the Right to Information Act in 2005 under the Government of India to act upon
complaints from those individuals who have not been able to submit information requests to a Central
Public Information Officer or State Public Information Officer due to either the officer not have been
appointed, or because the respective Central Assistant Public Information Officer or State Assistant
Public Information Officer refused to receive the application for information under the Right to
Information Act.
Option 4 is correct. Election commission: It is constitutional bodies that mainly function for the conduct,
control, supervise the election. It also performs judicial function e.g. determination of disqualification of
Member of legislator or examining the violation of model code of conduct. Hence, it is a quasi-judicial
body.
Source: Laxmikanth
https://blog.forumias.com/answered-what-is-a-quasi-judicial-body-explain-with-the-help-of-
concrete-examples/

Q.90)
Ans) a
Exp) Option a is the correct answer.
Mangroves are the most important and productive ecosystem on the earth. The root system of
mangroves stabilizes the coastline, reduces soil erosion from storm surge and provides a favourable
environment and nursery ground for fishes, crabs, and other aquatic organisms.
Statement 1 is incorrect: The Sustainable Aquaculture in Mangrove Ecosystem (SAIME) initiative is being
launched by the Non-Governmental Organisation (NGO) Nature Environment and Wildlife Society
(NEWS) in association with Global Nature Fund (GNF) and Naturland Bangladesh Environment and
Development Society (BEDS).
Statement 2 is correct: The SAIME initiative aims to curb the practice of unsustainable aquaculture,
particularly shrimp collection, after cleaning large tracts of mangrove forests in Sunderbans. Under the
SAIME initiative, farmers are planting mangrove trees around shrimp ponds to protect both mangrove
ecosystems as well as improve the productivity of shrimps. Hence this initiative promotes the wise use of
the mangrove ecosystem to satisfy communities’ livelihood needs, is correct.
Statement 3 is incorrect: The Sustainable Aquaculture in Mangrove Ecosystem (SAIME) initiative has
been implemented only in the state of West Bengal to protect the Mangrove ecosystem in this area.
Under SAIME initiative, farmers have taken up cultivation of shrimp at 20 hectares at Chaital in West
Bengal’s North 24 Parganas, and 10 hectares at Madhabpur in adjoining South 24 Parganas.

Forum Learning Centre: Delhi - 2nd Floor, IAPL House, 19 Pusa Road, Karol Bagh, New Delhi - 110005 | Patna - 2nd floor, AG Palace, E Boring Canal
Road, Patna, Bihar 800001 | Hyderabad - 1st & 2nd Floor, SM Plaza, RTC X Rd, Indira Park Road, Jawahar Nagar, Hyderabad, Telangana 500020
9311740400, 9311740900 | https://academy.forumias.com | admissions@forumias.academy | helpdesk@forumias.academy
Page 42 of 47

PTS 2024 | B4 | L1 Test 1 - Solutions |


Source: https://naturewildlife.org/saime/
https://www.thehindu.com/news/national/other-states/sustainable-shrimp-cultivation-provides-
hope-for-mangrove-restoration-in-sundarbans/article66324538.ece

Q.91)
Ans) b
Exp) Option b is the correct answer.
Article 25 of the Indian Constitution guarantees the Right to Freedom of Religion. It is a fundamental right
that provides individuals with the freedom to profess, practice, and propagate their religion of choice.
Statement 1 is correct: Article 25 of the Indian Constitution covers not only religious beliefs and
doctrines but also religious practices and rituals. It guarantees individuals the freedom to not only hold
religious beliefs but also to practice and propagate their religion. This includes the freedom to engage in
religious ceremonies, rituals, and observances associated with their faith. The protection provided by
Article 25 extends to both the internal aspects of religion, such as beliefs and doctrines, as well as the
external manifestations, such as religious practices and rituals.
Statement 2 is incorrect: The rights under Article 25 of the Indian Constitution are subject to certain
restrictions, including public order, morality, and health. They are not explicitly subject to the
sovereignty and integrity of India. The restrictions on the freedom of religion are primarily aimed at
maintaining public order, protecting the interests of morality, and ensuring public health.
Statement 3 is correct: Under Article 25, State is permitted to: (a) regulate or restrict any economic,
financial, political or other secular activity associated with religious practice; and (b) provide for social
welfare and reform or throw open Hindu religious institutions of a public character to all classes and
sections of Hindus
Source: Laxmikanth Ch-7, Fundamental rights

Q.92)
Ans) d
Exp) Option d is the correct answer.
By the States Reorganisation Act (1956) and the 7th Constitutional Amendment Act (1956), the distinction
between Part A and Part B states was done away with and Part C states were abolished. Some of them
were merged with adjacent states and some other were designated as union territories. As a result, 14
states and 6 union territories were created on November 1, 1956. At present India has 28 states and 9 UTs.
Correct Chronological order is as under
In 1960, the bilingual state of Bombay was divided into two separate states–Maharashtra for Marathi
speaking people and Gujarat for Gujarati speaking people. Gujarat was established as the 15th state of
the Indian Union.
In 1966, the State of Punjab was bifurcated to create Haryana, the 17th state of the Indian Union, and the
union territory of Chandigarh.
In 2000, three more new States of Chhattisgarh, Uttarakhand and Jharkhand were created out of the
territories of Madhya Pradesh, Uttar Pradesh and Bihar, respectively. These became the 26th, 27th and
28th states of the Indian Union, respectively.
In 2014, the new state of Telangana came into existence. It was carved out of the territories of Andhra
Pradesh.
Source: CH-7 Laxmikant 6th Edition.pdf

Forum Learning Centre: Delhi - 2nd Floor, IAPL House, 19 Pusa Road, Karol Bagh, New Delhi - 110005 | Patna - 2nd floor, AG Palace, E Boring Canal
Road, Patna, Bihar 800001 | Hyderabad - 1st & 2nd Floor, SM Plaza, RTC X Rd, Indira Park Road, Jawahar Nagar, Hyderabad, Telangana 500020
9311740400, 9311740900 | https://academy.forumias.com | admissions@forumias.academy | helpdesk@forumias.academy
Page 43 of 47

PTS 2024 | B4 | L1 Test 1 - Solutions |

Q.93)
Ans) c
Exp) Option c is the correct answer.
According to Dr. B.R. Ambedkar, the Directive Principles of State Policy are a ‘novel feature’ of the
Indian Constitution. They are in the nature of general directions, instructions or guidelines to the State.
Directive Principles embody the aspirations of the people, objectives and ideals which Union and the
State governments must bear in mind while making laws and formulating policies.
Directive Principles are certain ideals, particularly aiming at socio-economic justice, which according to
the framers of the Constitution, Indian State should strive for. They seek to establish a ‘welfare state’ in
India.
Source: Indian Polity by Laxmikant – 6th Edition – Chapter 3 Salient Features of the Constitution.

Q.94)
Ans) d
Exp) Option d is the correct answer.
Democracy is of two types-direct and indirect. In direct democracy, the people exercise their supreme
power directly as is the case in Switzerland. There are four devices of direct democracy, namely,
Referendum, Initiative, Recall and Plebiscite. In indirect democracy, on the other hand, the
representatives elected by the people exercise the supreme power and thus carry on the government and
make the laws. This type of democracy, also known as representative democracy, is of two kinds-
parliamentary and presidential.
Pair 1 is correctly matched: Referendum is a procedure whereby a proposed legislation is referred to the
electorate for settlement by their direct votes.
Pair 2 is correctly matched: Initiative is a method by means of which the people can propose a bill to the
legislature for enactment.
Pair 3 is correctly matched: Recall is a method by means of which the voters can remove a representative
or an officer before the expiry of his term, when he fails to discharge his duties properly.
Pair 4 is correctly matched: Plebiscite is a method of obtaining the opinion of people on any issue of
public importance. It is generally used to solve the territorial disputes.
Source: M Lakshmikanth ch-4

Q.95)
Ans) a
Exp) Option a is the correct answer
Neuromorphic computers mimic the neuro-biological networks in the human brain and perform tasks
efficiently and effectively, such as visual recognition and data interpretations. Compared to traditional
computers, neuromorphic computers are built to work like a human brain and so their processing is
highly rapid.
Statement 1 is correct: It is true that Neuromorphic computers are modelled after systems in the
human brain and nervous system. Traditional computers have physically separated memory storage
and processing units, and the human brain is a supreme biological computer that is smaller and more
efficient due to the presence of a synapse (the connection between two neurons) that plays the role of
both processor and memory storage unit.

Forum Learning Centre: Delhi - 2nd Floor, IAPL House, 19 Pusa Road, Karol Bagh, New Delhi - 110005 | Patna - 2nd floor, AG Palace, E Boring Canal
Road, Patna, Bihar 800001 | Hyderabad - 1st & 2nd Floor, SM Plaza, RTC X Rd, Indira Park Road, Jawahar Nagar, Hyderabad, Telangana 500020
9311740400, 9311740900 | https://academy.forumias.com | admissions@forumias.academy | helpdesk@forumias.academy
Page 44 of 47

PTS 2024 | B4 | L1 Test 1 - Solutions |

Statement 2 is incorrect: Neuromorphic computing uses hardware based on the structures and
processes of neurons and synapses in biological brains (not DNA of humans). The most common form of
neuromorphic hardware is the spiking neural network (SNN).
Statement 3 is incorrect: Neurons use chemical and electronic impulses to send information between
different regions of the brain and the rest of the nervous system. Neuromorphic computers by using this
process, will make it function faster and conserve more energy as compared to traditional computers.
Statement is incorrect as Neuromorphic computers consume less energy than traditional computers.
Source: https://blog.forumias.com/artificial-synapse-developed-for-brain-like-computing-with-
industry-compatible-nitride-semiconductors/#Why_Neuromorphic_computing

Q.96)
Ans) c
Exp) Option c is the correct answer.
Statement 1 is incorrect. All the DPSP since the adoption and enforcement of the constitution have been
non-justiciable. The 25th amendment act provided that any law made to give effect to the Directive
Principles contained in Article 39 (b) or (c) cannot be challenged on the ground of violation of the rights
guaranteed by Articles 14, 19 and 31.
Statement 2 is correct. Though the Directive Principles are non-justiciable, the Constitution (Article 37.
makes it clear that ‘these principles are fundamental in the governance of the country and it shall be the
duty of the state to apply these principles in making laws’. Thus, they impose a moral obligation on the
state authorities for their application.
Statement 3 is correct. The 73rd Constitutional Amendment Act, 1992, implemented the constitutional
obligation stated in Article 40 (Organisation of village panchayats).
Source: Indian Polity by M. Laxmikanth 5th edition

Q.97)
Ans) a
Exp) Option a is the correct answer
The Kesavananda Bharati case (1973) introduced the doctrine of basic Structure to limit the amendment
power of parliament to prevent any changes in the constitution that may affect the basic values
enshrined in the Constitution.
Statement 1 is incorrect: Basic structure of the constitution ensures the supremacy of constitution in
India, not judicial supremacy. Basic structure doctrine was introduced by the Supreme court to prevent

Forum Learning Centre: Delhi - 2nd Floor, IAPL House, 19 Pusa Road, Karol Bagh, New Delhi - 110005 | Patna - 2nd floor, AG Palace, E Boring Canal
Road, Patna, Bihar 800001 | Hyderabad - 1st & 2nd Floor, SM Plaza, RTC X Rd, Indira Park Road, Jawahar Nagar, Hyderabad, Telangana 500020
9311740400, 9311740900 | https://academy.forumias.com | admissions@forumias.academy | helpdesk@forumias.academy
Page 45 of 47

PTS 2024 | B4 | L1 Test 1 - Solutions |

the excessive use of power by any organs of the state - legislature, executive or judiciary to uphold the
core provisions of the constitution of India.
Statement 2 is correct: It is true that bill amending the constitution cannot be introduced in the state
legislative assemblies. In fact, no constitutional amendment bill can be introduced in the state
legislative assembles.
Statement 3 is incorrect: Parliament can amend fundamental rights if such changes does not affect the
basic structure of the constitution. Thus, it is incorrect to say that basic structure prohibits parliament
from amending any of the fundamental rights. The Supreme court in Kesavananda Bharati case upheld
the validity of the 24th Constitutional Amendment Act (CAA) and held that parliament is empowered to
take away or abridge any of the Fundamental Rights, if such changes does not alter the basic structure of
the constitution.
Statement 4 is incorrect: High court can strike down laws/executive actions even if it violates non-
basic structure of the constitution. Article 226 of the Indian constitution gives the high court the power
to review legislative/executive actions which contravenes any of the constitutional provisions which
includes both basic and non-basic features of the constitution.
Source: Laxmikanth - Amendment of the constitution
Basic structure of the constitution

Q.98)
Ans) b
Exp) Option b is the correct answer.
Conciliation is a non-binding procedure in which an impartial third party, the conciliator assists the
parties to a dispute in reaching a mutually satisfactory agreed settlement of the dispute.
Other forms of Alternative Dispute Resolution methods are:
1) Arbitration – It is a procedure in which the dispute is submitted to an arbitral tribunal which makes a
decision on the dispute that is binding on the parties.
2) Mediation – It is a procedure in which the parties discuss their disputes with the assistance of a
trained impartial third person(s) who assists them in reaching a settlement.
3) Negotiation - It is a process where two parties in a conflict or disagreement try to reach a resolution
together.
Mediation is a process of resolving issues between party where third party assist them in resolving
dispute, while in conciliation method in which an expert is appointed to settle dispute between the
parties.
Source: https://www.legalserviceindia.com/legal/article-2810-conciliation-in-india.html

Q.99)
Ans) c
Exp) Option c is the correct answer.
Articles 1 to 4 under Part-I of the Constitution deal with the Union and its territory. Article 1 describes
India, that is, Bharat as a ‘Union of States’ rather than a ‘Federation of States’. This provision deals with
two things: one, name of the country; and two, type of polity. Article 2 relates to the admission or
establishment of new states that are not part of the Union of India
Options 1, 3, and 4 are correct: Article 3 deals with the internal re-adjustment inter se of the territories of
the constituent states of the Union of India.
Article 3 authorizes the Parliament to:
1) form a new state by separation of territory from any state or by uniting two or more states or parts
of states or by uniting any territory to a part of any state;

Forum Learning Centre: Delhi - 2nd Floor, IAPL House, 19 Pusa Road, Karol Bagh, New Delhi - 110005 | Patna - 2nd floor, AG Palace, E Boring Canal
Road, Patna, Bihar 800001 | Hyderabad - 1st & 2nd Floor, SM Plaza, RTC X Rd, Indira Park Road, Jawahar Nagar, Hyderabad, Telangana 500020
9311740400, 9311740900 | https://academy.forumias.com | admissions@forumias.academy | helpdesk@forumias.academy
Page 46 of 47

PTS 2024 | B4 | L1 Test 1 - Solutions |

2) increase the area of any state;


3) diminish the area of any state;
4) alter the boundaries of any state; and
5) alter the name of any state.
However, Article 3 lays down two conditions in this regard: one, a bill contemplating the above changes
can be introduced in the Parliament only with the prior recommendation of the President; and two,
before recommending the bill, the President has to refer the same to the state legislature concerned for
expressing its views within a specified period. The President (or Parliament) is not bound by the views of
the state legislature and may either accept or reject them, even if the views are received in time.
Option 2 is incorrect: The power of Parliament to diminish the area of a state (under Article 3) does not
cover cession of Indian territory to a foreign country. Hence, Indian territory can be ceded to a foreign
state only by amending the Constitution under Article 368. The Constitutional amendment act under
Article 368 does not require prior permission of the president.
Source: Laxmikanth Chapter 5, Union and its territories (article 3), page 138
Laxmikanth Chapter 10, Amendment of the Constitution, page 275

Q.100)
Ans) a
Exp) Option a is the correct answer.
Statement 1 is incorrect: Galathea National Park is located in the Union Territory of Andaman and
Nicobar Islands. It is located on the island of Great Nicobar in the Nicobar Islands in the Bay of Bengal,
whereas Rann of Kutch is located in the state of Gujarat, in the Arabian Sea.

Statement 2 is correct: It is true that Galathea National Park is the nesting site for the world's largest
turtle called leatherback turtle. Further the notable animal species found in the park are giant robber
crab, megapode and Nicobar pigeon.

Forum Learning Centre: Delhi - 2nd Floor, IAPL House, 19 Pusa Road, Karol Bagh, New Delhi - 110005 | Patna - 2nd floor, AG Palace, E Boring Canal
Road, Patna, Bihar 800001 | Hyderabad - 1st & 2nd Floor, SM Plaza, RTC X Rd, Indira Park Road, Jawahar Nagar, Hyderabad, Telangana 500020
9311740400, 9311740900 | https://academy.forumias.com | admissions@forumias.academy | helpdesk@forumias.academy
Page 47 of 47

PTS 2024 | B4 | L1 Test 1 - Solutions |

Statement 3 is incorrect: Recently Galathea Bay (an area surrounding Galathea NP) was in news due to
National Board for Wildlife (NBWL)’s decision to de-notify entire Galathea Bay for implementing the
Ministry of Ports, Shipping, and Waterways project called International Container Trans-Shipment Port
(ICTP) at Great Nicobar Islands. The National Park has not been chosen for the introducing Asiatic lion.
Source: https://www.hindustantimes.com/environment/denotify-turtle-nesting-site-in-andaman-for-
shipment-project-wildlife-board-101611807608329.html

Forum Learning Centre: Delhi - 2nd Floor, IAPL House, 19 Pusa Road, Karol Bagh, New Delhi - 110005 | Patna - 2nd floor, AG Palace, E Boring Canal
Road, Patna, Bihar 800001 | Hyderabad - 1st & 2nd Floor, SM Plaza, RTC X Rd, Indira Park Road, Jawahar Nagar, Hyderabad, Telangana 500020
9311740400, 9311740900 | https://academy.forumias.com | admissions@forumias.academy | helpdesk@forumias.academy

You might also like